ROSH: GI/OBGYN mix 2

Ace your homework & exams now with Quizwiz!

A 65-year-old woman presents to the clinic with constant abdominal pain in the left lower quadrant for the past two days. She also reports being slightly constipated recently. Vital signs are T 100.4°F, HR 83, BP 116/76 mm Hg, and RR 20. On abdominal exam, she is tender to palpation in the left lower quadrant but has no peritoneal signs or palpable masses. Which of the following is the most likely diagnosis? Acute appendicitis Acute diverticulitis Diverticular perforation Diverticulosis

Acute diverticulitis

A 29-year-old woman presents to deliver a full-term fetus. There are no concerns for medical or obstetrical conditions or complications. Her vital signs are stable and within normal limits. A vaginal examination is performed and the fetus is noted to have a longitudinal lie and brow presentation is determined by palpation of the anterior fontanelle. Which sutures intersect to form this fontanelle? Coronal, frontal, and sagittal Coronal, lambdoid, and sagittal Frontal and sagittal Lambdoid and sagittal

Coronal, frontal, and sagittal

A 32-year-old woman presents for a colonoscopy due to complaints of intermittent diarrhea and abdominal pain for years. On colonoscopy, there are patches of inflamed and ulcerated mucosa with a cobblestone appearance. Examination of the terminal ileum reveals similar patches of inflammation, but there appears to be no involvement of the rectal mucosa. Which of the following is the most likely diagnosis? Celiac disease Crohn disease Infectious colitis Ulcerative colitis

Crohn disease Crohn disease (CD) is a disorder characterized by periods of prolonged diarrhea with or without abdominal pain, weight loss, and fatigue. The etiology is unclear, but symptoms arise due to transmural inflammation, which can occur anywhere along the gastrointestinal tract from the mouth to the anus, although the ileum is the most common area of involvement. Symptoms often fluctuate over time, and clinical manifestations of the disease are variable, so patients may go years without an accurate diagnosis. Diarrhea can be caused by the malabsorption of bile salts or water in the small intestine, by steatorrhea that occurs as bile salts are depleted in the body, or by bacterial overgrowth in the gut. The diarrhea can be accompanied by gross or microscopic bleeding, although gross bleeding is less common than in ulcerative colitis. Crampy abdominal pain is a common complaint and often accompanies fibrotic strictures, which can lead to repeated episodes of small bowel or colonic obstruction. Another complication of Crohn disease is the formation of sinus tracts and fistulas, whether to the bladder, vagina, skin, or other segments of bowel. The formation of sinus tracts can also cause the development of phlegmon, walled-off inflammatory processes without infection, which can usually be palpated on physical exam. Perianal disease is very common and may present as anorectal abscesses or fistulas, large perianal skin tags, or anal fissures. Normally, the distal ileum is involved in the absorption of fat with the help of bile salts. As this process can become significantly impaired in Crohn disease, severe symptoms of malabsorptioncan occur including clotting abnormalities, osteomalacia, and hypocalcemia. Aphthous ulcers may develop in the mouth, and the esophagus and stomach may become involved giving rise to symptoms such as dysphagia, odynophagia, epigastric pain, nausea, and vomiting. Systemic symptoms are very common in patients with Crohn disease and include fatigue, weight loss, arthritis, arthropathy, eye and skin involvement, primary sclerosing cholangitis, venous and arterial thromboembolism, renal stones, vitamin and mineral deficiencies, and pulmonary involvement. The initial evaluation for a patient suspected of having Crohn disease should include CBC, liver enzymes, renal function tests, electrolytes, blood glucose, erythrocyte sedimentation rate (ESR), C-reactive protein (CRP), serum iron, vitamin D, vitamin B12, and fecal culture and ova and parasites if diarrhea is present. CRP levels are generally thought to correlate with disease activity, and normalization of CRP after commencement of therapy indicates success. CRP can also be utilized in differentiating Crohn disease from ulcerative colitis, as it is usually higher in CD. Colonoscopy can by used to diagnose ileocolonic Crohn disease. Typical findings include skipped areas of involvement adjacent to normal appearing mucosa with rectal sparing. Affected areas will be ulcerated and inflamed with a polypoid or cobblestone appearance. A large majority of patients with Crohn disease have small bowel involvement, and therefore, additional imaging is usually necessary because it is not accessible to traditional endoscopy. Wireless capsule endoscopy, in which a patient swallows a small pill-shaped camera, is increasingly being used to assess the small bowel. It is contraindicated if the patient is suspected of having a stricture that would impede its passing. Other imaging modalities used to assess the small bowel include upper gastrointestinal series with small bowel follow-through (includes ingestion of a barium solution and subsequent imaging), computed tomography, computed tomography enterography, magnetic resonance imaging, and magnetic resonance enterography. Most patients will experience periods of exacerbation and periods of remission. Many patients will at some point require hospitalization and surgical resection due to intractable disease. There are many pharmacological options for the treatment of Crohn disease, and the choice of therapy will depend on the affected anatomic location, the severity of disease, and whether the goal is to induce or maintain remission. The classes of medications include oral 5-aminosalicylates, glucocorticoids, immunomodulators, and biologic therapies.

Which of the following is the appropriate gestational age to screen pregnant women for Group B streptococcus infection? 10 weeks 20 weeks 28 weeks 36 weeks

36 weeks

A 41-year-old woman presents to her gynecologist for a routine check-up. She was recently married and is now interested in having children but wonders if she needs to find an egg donor. She has never been pregnant before, but she has always had protected intercourse. She has regular menses. Which of the following is the best test to screen for her ovarian reserve? Anti-müllerian hormone level Hysterosalpingogram Luteinizing hormone level Mid-luteal phase serum progesterone level

Anti-müllerian hormone level It is well understood that fertility declines with increasing female age. Spontaneous conceptions with healthy outcomes are the exception after age 43. One of the key biological reasons for diminishing fertility with maturity in women is the finite quantity of (fixed at birth) and declining quality of oocytes. For many societal reasons the average age at first conception is increasing. As patients present for consultation about fertility later in their reproductive years, tests of reproductive potential have become an increasingly important component of the evaluation of female fertility, and evaluation for infertility is advised immediately for women over 40 years oldwho are seeking to conceive. The term ovarian reserve describes the functional capacity of the ovaries, including the number and quality of remaining oocytes. A number of diagnostic tests are employed to ascertain oocyte quality, quantity, and overall reproductive potential. There is currently no one best test for ovarian reserve, but there are several which, when used in combination, provide a good assessment. The most commonly used tests are anti-mllerian hormone (AMH), antral follicle count, clomiphene citrate challenge test, and cycle day 3 follicle-stimulating hormone or estradiol. Of these, the AMH is easiest to obtain and may be the best overall test. AMH is secreted by preantral and early antral follicles, overall reflecting the size of the patients remaining follicle reserve. It can be measured any time during the menstrual cycle. AMH levels decline steadily with age and will be undetectable at menopause. For younger women less than 35 years of age, evaluation of infertility is typically undertaken after one year of unprotected intercourse and after six months of unprotected intercourse in women 35 years and older. Assessment should begin with a thorough history and physical examination of both partners in an infertile coupling. The evaluation of infertility can be an emotional and stressful one, so clinicians should exercise sensitivity and take a counseling approach to their interactions with patients. When possible and applicable, both partners should be counseled and evaluated together. Evaluation of female infertility should consider three components that may affect outcomes: (1) ovulatory function, (2) anatomical factors, and (3) ovarian reserve. Ovulatory function can usually be assessed by history of regular menses approximately every four weeks and molimina symptoms (such as bloating, breast tenderness, fatigue). Diagnostic tests like a mid-luteal phase serum progesterone level or over-the-counter ovulation prediction tests can confirm recent ovulation. A progesterone level 5 ng/mL indicates that ovulation is occuring. In this case, patency of the fallopian tubes and assessment of the ovarian cavity may be confirmed by hysterosalpingogram or hysterosalpingo-contrast sonography. Surgical correction of adhesions or tubal obstruction may be performed and the need for reproductive technology should be evaluated. Women found to have progesterone levels < 5ng/mL are considered non-ovulatory, and should have TSH, prolactin, FSH, and estradiol levels measured to rule out underlying causes. Underlying causes should be treated and ovulation may be induced with clomiphene.

A 30-year-old woman and her husband present to their reproductive endocrinologist for evaluation of infertility. They have been trying regularly for the last 18 months to get pregnant. They have undergone initial evaluations, including history, physical exam, semen analysis, and laboratory testing, which have led to a diagnosis of anovulation due to polycystic ovarian syndrome. They have returned to discuss treatment approaches to achieve their goal of pregnancy. Which of the following is the best initial treatment for this patient? Clomiphene Gonadotropin-releasing hormone Letrozole Metformin

Clomiphene Infertility is defined as the inability to conceive after 12 months of normal, regular, unprotected sexual activity. Male causes of infertility include primary hypogonadism, disordered sperm transport, and secondary hypogonadism. However, half of male causes are idiopathic. Female causes are more commonand include amenorrhea, endometriosis, ovulatory dysfunction, and tubal defects. Of these, the most common causes for female infertility are amenorrhea and ovulatory dysfunction, whose etiology may be more specifically defined as hypothalamic/pituitary, polycystic ovarian syndrome, premature ovarian failure, or uterine or outflow tract disorders. Diagnosis of the etiology of infertility includes a detailed history and physical exam of the woman and semen analysis of the man. Measurements of thyroid-stimulating hormoneand prolactin in both the man or woman and follicle-stimulating hormone and androgens in the woman are helpful along with karyotype when genetic causes are suspected. If other testing is unrevealing, ovarian sonogram, hysterosalpingogram, and endometrial biopsy may be indicated. Treatment depends on the suspected etiology of infertility. Most women with infertility will fall into the etiologic category of normogonadotropic normoestrogenic anovulation. These women usually respond well to medically induced ovulation. The first-line treatment for this purpose is clomiphene, a selective estrogen receptor modulatorthat increases gonadotropin release by acting as both an estrogen antagonist and agonist. Clomipheneappears to act primarily on the hypothalamus where it blocks the negative feedback effect of endogenous estradiol, leading to an increase in frequency of hypothalamic gonadotropin-releasing hormone release and subsequently increases follicle-stimulating hormone and luteinizing hormone. Clomiphene is metabolized through the liver and excreted through the feces. There is no evidence linking the use of clomiphene to congenital malformations. Once ovulatory dysfunction is established and patients have been educated on the management approach, clomiphene can typically be started on the fifth day of a cycle. The starting dose is 50 mg daily for five days. The dose may increase by increments of 50 mg up to a maximum daily dose of 150 mg to achieve ovulation. The response to treatment can be monitored by determination of luteinizing hormone surge by urinary luteinizing hormone kits. A luteinizing hormone surge typically occurs five to 12 days after administration of clomiphene, and ovulation occurs within 48 hours of the surge. This confirmation assists with the timing of intercourse during a cycle as the window of highest fertility is usually the three days, including the luteinizing hormone surge and two days after. Measuring basal body temperature and tracking conversion to a biphasic basal temperature suggesting ovulation is a less expensive but also less precise method of confirming that ovulation has been induced. Once induction of ovulation is confirmed, the same dose of clomiphene used to achieve ovulation can be continued for four to six cycles.

An 80-year-old man with a history of moderate dementia is brought in by ambulance to the emergency department from his long-term care facility for abdominal pain and distension for the past hour. His caregiver is present and states that he has not had a bowel movement for several days. Physical exam reveals a largely distended abdomen that sounds hollow with percussion. Abdominal radiograph was obtained and can be seen above. Which of the following is the most likely diagnosis? Colonic volvulus Ileus Ischemic colitis Toxic megacolon

Colonic volvulus Colonic volvulus is a life-threatening condition that occurs when a part of the colon twists upon its mesenteryresulting in colonic obstruction or vascular compromise. Delaying treatment is associated with high morbidity and mortality. Twisting can occur in any position of the large bowel but most commonly involves the sigmoid colon due to the mesenteric anatomy. Common predisposing factors include chronic constipation, presence of a pelvic or ovarian mass, and pregnancy. Patients with volvulus are likely to be elderly, debilitated, and bedridden,as well as have associated dementia or psychiatric impairment. Postoperative adhesions, hernias, carcinoma, and intussusception are less common risk factors. Those who have had volvulus are likely to have a history of acute episodes in the past that have spontaneously resolved. Patients will present with acute onset of colicky abdominal pain, cramping, abdominal distension, obstipation, and constipation. As the condition progresses, nausea and vomiting will occur. Persistent abdominal pain is a sign that there is development of colonic obstruction, which can lead to ischemia, gangrene, and bowel perforation. Physical exam will reveal a largely distended, tympanitic abdomen. Patients who have rebound tenderness should be suspected to have peritonitis due to a perforated bowel. A complete blood count with differential may reveal an elevated white blood cell count, which would point to bowel ischemia, peritoneal infection, or sepsis. An abdominal radiograph can confirm diagnosis by revealing U-shaped, distended sigmoid colon ("bent inner tube" sign). Other studies such as computed tomography scan, barium enema, and colonoscopy or sigmoidoscopy may be helpful but not necessary if the radiograph is sufficient for diagnosis. Definitive treatment includes decompression via sigmoidoscopy and surgical resection. Patients who have peritonitis or ischemic bowel will require fluid resuscitation and broad-spectrum antibiotic treatment with emergency surgery to follow.

A 22-year-old woman presents to the clinic with purulent vaginal discharge and bleeding after intercourse. Pelvic examination reveals purulent discharge on the ectocervix and cervical friability. She is afebrile and has no cervical motion tenderness on exam. Which of the following is the most likely diagnosis? Cervicitis Pelvic inflammatory disease Ruptured ovarian cyst Vaginitis

Cervicitis Cervicitis is an inflammatory disorder that primarily affects the endocervical canal glands. Sexuallytransmitted infections, most frequently gonorrhea and chlamydia, are the most common cause of acute cervicitis. Less common infectious causes include trichomoniasis (Trichomonas vaginalis), bacterial vaginosis, and herpes simplex virus. Irritation from foreign bodies is an example of a noninfectious etiology. Infectious cervicitis is clinically significant because it can ascend and cause pelvic inflammatory disease, can be transmitted to sexual partners, and can cause pregnancy or neonatal complications. Most women with cervicitis are asymptomatic. Symptoms are often nonspecific when present and may include purulent or mucopurulent vaginal dischargeand intermenstrual or postcoital bleeding. Less common symptoms include dysuria, urinary frequency, dyspareunia (pain with intercourse), and vulvovaginal irritation. Pain and fever are atypical findings and suggest pelvic inflammatory disease or herpes simplex virus infection. Pelvic exam findings may include purulent discharge on the ectocervix, cervical friability (minor trauma precipitating bleeding), and erythema or edema of the cervix. Trichomoniasis may present with punctate hemorrhages on the vagina or cervix, which is also called strawberry cervix. Trichomoniasis may also manifest with green-yellow frothy, malodorous discharge.Herpes simplex virus classically causes painful vesicular lesions on an erythematous base. Gonorrhea and chlamydia do not have specific clinical findings and are either asymptomatic or present with indistinguishable symptoms, such as mucopurulent discharge. Cervical motion tenderness suggests pelvic inflammatory disease. The diagnosis of acute cervicitis is clinical and based on the presence of purulent or mucopurulent discharge and endocervical friability. Patients who are diagnosed with acute cervicitis should be tested for chlamydia, gonorrhea, trichomoniasis, and bacterial vaginosis. The preferred method for gonorrhea, chlamydia, and trichomoniasis testing is nucleic acid amplification testing (NAAT). Herpes simplex virus can be diagnosed with viral culture or polymerase chain reaction of vesicular fluid from viral lesions. Nucleic acid amplification testing can be performed on a swab of vaginal fluid, on a sample obtained by rotating a swab within the endocervical canal, or on a urine sample in men. The goals of treatment of acute cervicitis include relieving symptoms, preventing pelvic inflammatory disease, and reducing transmission to sexual partners. Patients who are diagnosed with acute cervicitis should be treated with empiric antibiotics. Since gonorrhea and chlamydia are the most common causes, the empiric antibiotic regimen should cover at least these two causes. The recommended antibiotic regimen to cover gonorrhea and chlamydia infection consists of ceftriaxone 500 milligrams administered intramuscularly and azithromycin 1 gram given orally in pregnancy or doxycycline (as a course) in nonpregnant patients. If herpes simplex virus infection is suspected, then acyclovir or valacyclovir is the treatment of choice. Bacterial vaginosis is treated with oral or topical clindamycin or metronidazole. Trichomoniasis is treated with oral metronidazole or tinidazole. All sexual partners of individuals with chlamydia, gonorrhea, or trichomoniasis should be treated for the same infection that was diagnosed in their sex partner. Patients and their sex partners should abstain from sexual intercourse until therapy is completed. All patients being evaluated for gonorrhea, chlamydia, or trichomoniasis should be offered testing for human immunodeficiency virus (HIV) and syphilis.

A 43-year-old woman presents to the emergency department complaining of sharp abdominal pain, nausea, and vomiting. Physical exam reveals tachycardia and abdominal distention with high-pitched "tinkling" sounds heard on auscultation of the abdomen. Findings on X-ray indicate a mechanical small bowel obstruction. Which of the following selections from her past medical history is most pertinent to her current condition? Cesarean section History of smoking Prior radiation therapy Recent history of obstipation

Cesarean section Small bowel obstruction is the disruption of the normal intraluminal flow of contents through the small intestine. Itcan be due to functional or mechanical obstruction and can be acute or chronic in nature. The most common etiologies of small bowel obstruction are adhesions secondary to prior abdominal or pelvic surgery,including cesarean section, and hernias. Other risk factors include intestinal inflammation, history of or increased risk for neoplasm, prior radiation therapy, or foreign body ingestion. Presentation of small bowel obstruction is often acute with nausea, vomiting, and abdominal pain being common complaints. Obstipation,which is the inability to pass stool or flatus, may also be a presenting symptom. The hallmark sign of small bowel obstruction is dehydration, which is evidenced during examination as tachycardia, orthostatic hypotension, reduced urine output, or dry mucous membranes. Abdominal distention is common, and high-pitched "tinkling" is classically described on abdominal auscultation. Diagnosis is aided initially by plain radiograph, which shows dilated loops of bowel with air-fluid levels. A "string of beads" can sometimes be found on upright or lateral X-ray. Abdominal CT is more useful in a nonemergent setting to further evaluate the location and severity of obstruction. Treatment of small bowel obstruction depends on the severity of obstruction and consists of surgical intervention with adhesiolysis or conservative nonoperative management (preferred) including bowel rest, nasogastric tube and intravenous fluids. Advanced small bowel obstruction can cause intestinal ischemia due to excessive bowel dilatation and potential strangulation. These complications disrupt perfusion of the small bowel and the resultant necrosis and perforation increase the overall mortality of this condition. Surgical evaluation with intestinal resection is necessitated in patients with complications to remove the damaged small intestine. These patients are at an increased risk for post-surgical adhesions, thereby increasing their chance of recurrence, as well as gastrointestinal complications such as malabsorption due to decreased bowel surface area.

Endoscopic retrograde cholangiopancreatography (ERCP) is indicated for the treatment of what disease process? Cholangitis Cholecystitis Cholelithiasis Functional gallbladder disorder

Cholangitis Cholangitis is the simultaneous obstruction and infection of the biliary tree. Also known as ascending cholangitis or suppurative cholangitis, this disease process can range from mild to life-threatening. The most common cause of acute cholangitis is choledocolithiasis, or biliary stones in the common bile duct. Other causes include biliary strictures (congenital, post-infectious, or inflammatory), and malignancy (of the gallbladder, bile duct, ampulla, duodenum, or the pancreas). It is more common in people of northern European descent with fair skin, as well as Hispanic, American Indian, and Pima Indian populations. Risk factors include having undergone ERCP, infection with HIV, acute pancreatitis, and infection with certain parasites (liver flukes, Ascaris roundworm). The normal mechanisms that maintain the sterility of the biliary tree include the sphincter of Oddi, the flushing activity of bile, and the bacteriostatic properties of bile salts. When there is obstruction of the common bile duct, bacteria from the duodenum are able to ascend the biliary tree, causing infection and possibly septicemia. Additionally, biliary obstruction will cause increased biliary pressure, which leads to increased permeability of bile ductules and the translocation of bacteria from the portal circulation into the biliary tract. The most common causative organisms include Escherichia coli, Klebsiella, and Enterobacter species. Patients classically present with Charcot triad: fever, abdominal pain, and jaundice. Between 50-75% of patients will present with all three findings. As symptoms progress and patients deteriorate, they may also present with Reynolds pentad, which includes Charcot triad plus hypotension and mental status changes. Additionally, they may also exhibit symptoms related to septicemia, multiple organ system dysfunction, and shock. Laboratory abnormalities typically include leukocytosis and a cholestastic pattern of liver test abnormalities, with elevations in serum alkaline phosphatase, gamma-glutamyl transferase (GGT), and bilirubin (primarily conjugated). Additionally, both alanine aminotransferase (ALT) and aspartate aminotransferase (AST) may be elevated. Given the potential life-threatening course this disease process can take, a high clinical suspicion must be maintained. Therefore, in patients with Charcot triad and liver test abnormalities, patients often proceed directly to endoscopic retrograde cholangiopancreatography (ERCP) for diagnosis and treatment. This procedure involves the use of a specialized upper endoscope with a side-view, through which wires and instruments can be passed to remove stones, inject contrast, or to perform other procedures in the ampulla, bile duct, or pancreatic duct. If the diagnosis is unclear, diagnostic imaging should be pursued before proceeding with ERCP. Options include abdominal ultrasound, abdominal CT, magnetic resonance imaging (MRI), or magnetic resonance cholangiopancreatography (MRCP). MRCP is reserved for cases when the diagnosis is still unclear after ultrasound and CT imaging. Diagnosis of acute cholangitis is made when the patient has evidence of systemic inflammation (fever, leukocytosis, elevated C-reactive protein), evidence of cholestasis on laboratory tests, and imaging showing biliary dilatation, stone, stent, or stricture. Treatment includes immediate empiric treatment with broad-spectrum antibiotics (piperacillin-tazobactam, a combination of metronidazole and a cephalosporin or a fluoroquinolone, or a carbapenem), supportive measures (intravenous hydration, analgesics), and biliary decompression. Biliary decompression is usually done via ERCP within 24 to 48 hours of presentation. If this is not possible or decompression is not achieved during ERCP, percutaneous transhepatic cholangiography or surgical decompression can be performed. For patients with severe cholangitis (considered severe if there is any evidence of cardiovascular, neurological, renal, respiratory, hepatic, or hematologic dysfunction) or who do not respond to initial treatment with antibiotics, ERCP should be performed urgently (within 24 hours). For patients with mild to moderate cholangitis (leukocytosis, fever, age > 75, hyperbilirubinemia, hypoalbuminemia), ERCP should be performed between 24-48 hours. The choice of antibiotics should take into account the patient's age, comorbidities, and whether the infection is community or hospital acquired. Once blood culture results are known, antibiotic therapy can be individualized. In addition to antibiotic treatment and decompression, addressing the underlying cause is important to prevent recurrence. For patients with cholelithiasis, elective cholecystectomy is recommended at a later time. Patients with benign biliary strictures may require surgical or endoscopic repair. Cholangitis secondary to malignancy may require the placement of a temporary stent of the common bile duct during endoscopic decompression.

Which of the following is a contraindication to mifepristone use in a patient needing first-trimester medication abortion? Asthma Chronic adrenal failure Nexplanon implant Nicotine dependence

Chronic adrenal failure Chronic adrenal failure and long-term corticosteroid use are contraindications to mifepristone use as the drug is a glucocorticoid receptor antagonist. Induced abortion is the termination of pregnancy through the use of medication or surgery. Medical abortion is performed with mifepristone in conjunction with misoprostol. Depending on gestational age or incomplete medical abortion, surgical abortion by dilation and curettage or dilation and evacuation may be performed up to 24 weeks from last menstrual period. Mifepristone can be used to terminate intrauterine pregnancy up to 70 days of gestation. Ultrasound should be used if gestational age is uncertain or if ectopic pregnancy is suspected. Mifepristone is contraindicated in chronic adrenal failure, hemorrhagic disorders or anticoagulant use, ectopic pregnancy, allergy to the medication, inherited porphyrias, or intrauterine device. Protocol for medical abortion includes mifepristone 600 mg orally followed by misoprostol 400 mcg orally 48 hours later or mifepristone 200 mg orally followed by misoprostol 800 mcg buccally 24-72 hours later. Follow-up with prescribing provider should be performed within two weeks following medication administration to ensure the effectiveness of treatment and to address any complications. Side effects of medications include abdominal pain, prolonged vaginal bleeding, gastrointestinal upset, fever, infection, sepsis, and death.

A 45-year-old woman with a recent diagnosis of diabetes mellitus presents to her primary care provider with the complaint weakness, diarrhea, and weight loss. She states that she has had nausea and epigastric pain. The patient appears jaundiced. Physical exam reveals a palpable mass in the right upper quadrant of her abdomen. A 4 cm pancreatic neoplasm at the head of the pancreas is seen on computed tomography. Which of the following is the most clearly established environmental risk factor for the development of pancreatic cancer? Cigarette smoking Diabetes mellitus Heavy alcohol use Nonsteroidal anti-inflammatory use

Cigarette smoking Pancreatic cancer is a broad term that usually refers to ductal adenocarcinoma of the pancreas. It is most commonly seen in elderly (>60 years old) and more common in African Americans. The most clearly established risk factor for development of pancreatic cancer is cigarette smoking. Obesity, heavy alcohol use, and physical inactivity have been associated with the development of pancreatic cancer as well. Patients will most commonly present with weight loss, steatorrhea, weakness, and epigastric pain, which radiates to the sides or straight through to the back. Epigastric pain is usually worse at night and improves when lying in a curled or fetal position. Jaundice and hepatomegaly are common signs of the condition. Jaundice occurs due to obstruction of the common bile duct by a mass in the head of the pancreas and my be accompanied by pruritus, darkening of the urine, and pale stools. Unexplained superficial thrombophlebitis, which may be migratory (classic Trousseau's syndrome), is sometimes present and reflects the hypercoagulable state that frequently accompanies pancreatic cancer. A recent onset of atypical diabetes mellitus may be noted as well. Physical exam may also reveal a painless palpable gallbladder (Courvoisier sign), ascites, left supraclavicular lymphadenopathy (Virchow's node), and a palpable periumbilical mass (Sister Mary Joseph's node). Common sites for metastases include the liver, peritoneum, lungs, and, less frequently, bone. All patients presenting with jaundice or epigastric pain should have an assay of serum aminotransferases, alkaline phosphatase, and bilirubin to determine if cholestasis is present. In addition, patients with epigastric pain should be evaluated for acute pancreatitis with a serum lipase. For patients with jaundice, initial imaging is usually a transabdominal ultrasound (US). US is highly sensitive for detecting biliary tract dilation, establishing the level of obstruction, and detecting pancreatic masses >3 cm. Abdominal computed tomography (CT) is the preferred initial imaging test in patients presenting with epigastric pain and weight loss that raise suspicion for pancreatic cancer. ERCP is the most sensitive test for diagnosing cancer of the pancreas and provides an opportunity to collect tissue samples for histologic diagnosis. However, ERCP is an invasive procedure and is associated with complications such as pancreatitis, bleeding, and cholangitis. Pancreatic cancer has a poor long-term prognosis as it usually presents relatively late in the disease process. Surgical resection (Whipple procedure) is the only potentially curative procedure available, but most patients are not candidates for the procedure due to the progression of the disease when diagnosed. If the tumor is unresectable and biliary obstruction is present from pancreatic cancer, ERCP with stent placement may be performed for biliary decompression.

Which of the following is the most appropriate therapy to treat premenstrual syndrome? Citalopram Ibuprofen Venlafaxine Vitamin B6

Citalopram Premenstrual syndrome (PMS) is a medical condition that occurs each luteal phase of a woman's menstrual cycle and is defined by physical, behavioral, and psychological symptoms that interfere with daily activities. Physical symptoms of PMS include headaches, bloating, and breast pain, while the psychological or behavioral manifestations of PMS present as irritability, anxiety, or depression. Severe symptoms of PMS include anger, severe irritability, and internal tension, changing the diagnosis from PMS to premenstrual dysphoric disorder. PMS is very common, with up to 90% of women experiencing symptoms at some point in their lifetime. There are five categories of PMS based on symptoms: anxiety, cravings, depression, hydration, and other or nonspecific. Diagnosis is based on clinical history after tracking symptoms and the timing of menses for two cycles. Physical exam findings may include edema of the lower extremities or breasts and mastalgia, although generally, the physical exam is noncontributory to the diagnosis. Lifestyle modifications, such as regular physical exercise and stress reduction techniques, should be advised for all patients with PMS, and these may be the only interventions required for patients with mild symptoms. When medication is indicated, first-line treatment is with a selective serotonin reuptake inhibitor (SSRI). Options include citalopram, escitalopram, fluoxetine, sertraline, or paroxetine, and dosing may be continuous throughout the cycle or during the luteal phase only.

A 46-year-old man presents with intermittent upper abdominal pain for the past several months. He states that the pain is cramping in nature and most commonly occurs a few hours after eating. A urea breath test is positive. The patient has no known allergies and is not on any current medication. Which of the following is the best therapy in a locale without widespread antibiotic resistance? Bismuth subsalicylate, tetracycline, and amoxicillin Clarithromycin, amoxicillin, and omeprazole High-dose amoxicillin and pantoprazole Levofloxacin, amoxicillin, and omeprazole

Clarithromycin, amoxicillin, and omeprazole Peptic ulcer disease is common in the general population. It is defined by a deep defect in the gastrointestinal mucosa. The two most common causes are a bacterial infection with Heliobacter pylori or the chronic use ofnonsteroidal anti-inflammatory (NSAIDs) medications. Risk factors for developing a peptic ulcer include smoking, the use of NSAIDs with other drugs that may create gastrointestinal irritation, and the use of clopidogrel. Symptoms most commonly include upper abdominal pain or dyspepsia. The pain is typically related to food intake. It is either made worse with food intake (i.e., pyloric channel ulcer) or made better with food (i.e., duodenal ulcer). Complications include bleeding, perforation, fistulization, or gastric outlet obstruction. Patients with a bleeding ulcer may present with coffee-ground emesis or black, tarry stools. Occasionally, there are chronic low levels of blood loss that can lead to an iron deficiency anemia. Endoscopycan be done for definitive diagnosis and to characterize the ulcerative lesion better. Typically, lesions appear well-circumscribed, smooth, and round. All patients with peptic ulcer disease should be tested for H. pylori infection. In patients with a positive H. pylori test, therapy is aimed at eradicating the infection. Common regimens include clarithromycin-based triple therapy or bismuth quadruple therapy. Treatment should be based on local rates of macrolide resistance and the presence of a penicillin allergy. Risk factors fortreatment failure include poor patient compliance and antibiotic resistance. For patients who have a negative H. pylori test or have NSAID-induced ulcerations, a proton pump inhibitor is used as first-line treatment.

A 45-year-old woman with a one-month history of epigastric pain, nausea, and vomiting presents to her primary care physician for follow-up of recent tests. Her stool antigen assay is positive for Helicobacter pylori infection. She has been otherwise healthy prior to this problem with no recent antibiotic use, and she has no known allergies to medications. Local antibiograms show favorable results for eradication using first-line treatment. Which of the following is the best approach for initial management? Bismuth quadruple therapy Clarithromycin-based triple therapy with amoxicillin Clarithromycin-based triple therapy with metronidazole Low-dose ranitidine

Clarithromycin-based triple therapy with amoxicillin Acute gastritis is an inflammatory process associated with a gastric mucosal disorder. The most common causes of gastritis are Helicobacter pylori infection and autoimmune causes. The use of nonsteroidal anti-inflammatory drugs and alcohol consumption are also commonly cited causes, although they are more correctly described as gastropathy. The characteristic symptoms of acute H. pylori infection include epigastric pain, nausea, and vomiting without fever. Serology and stool antigen assay are reliable, noninvasive tests to establish a diagnosis of H. pylori infection. Additional investigation via gastric mucosal biopsy is able to distinguish between acute, chronic active, chronic gastritis, and gastropathy. Immunologic testing should also be pursued if an autoimmune gastritis is suspected. Treatment for H. pylori infection is based on risk factors formacrolide resistance and history of penicillin allergy. For a macrolide-naïve patient, with no concern for local resistance to clarithromycin with no penicillin allergy, the recommended initial treatment for acute gastritis due to H. pylori is clarithromycin-based triple therapy with amoxicillin (clarithromycin, amoxicillin, and a proton-pump inhibitor) given twice daily for 14 days.

A 23-year-old woman presents to the office for follow-up on stomach pain and bowel irregularities. Celiac screening via serum and biopsy were negative. Despite the negative results, she has noticed that when she eliminates gluten from her diet she has significant symptom improvement. Which of the following is the best recommendation for the patient? Continue to consume gluten since celiac screening is negative Eliminate gluten Eliminate lactose Repeat testing in six months

Eliminate gluten Celiac disease is an autoimmune-mediated disease of the small intestine. The disease results in the stripping of intestinal cilia and produces a malabsorption. It is different from food allergies that are immune modulated with IgE or IgG which can produce anaphylaxis. Signs and symptoms may include chronic or recurrent diarrhea, malabsorption, weight loss, abdominal distension or bloating, unexplained iron deficiency anemia, or folate deficiency. When there is clinical suspicion of celiac disease in a patient over the age of two, the workup should begin with IgA anti-tissue glutaminase antibody and IgA endomysial antibody serum testing. Patients with true autoimmune disease will have a positive tissue biopsy of the small bowel. Histology is the way to distinguish between celiac disease and non-celiac gluten sensitivity. Non-celiac gluten sensitivity can create gastrointestinal distress similar to those with celiac disease, however, they are not at risk for nutritional deficiency and malabsorption. Patients with non-celiac gluten sensitivity will have negative serology, negative biopsy, and negative wheat allergy skin prick test but will benefit symptomatically from a gluten-free diet. Therefore, these patients may be advised to eliminate gluten from their diet.

A 48-year-old obese perimenopausal woman presents to the clinic for evaluation of abnormal uterine bleeding. She has a history of polycystic ovarian syndrome. Which of the following is the gold standard for diagnosis of endometrial hyperplasia? Endometrial sampling Pelvic ultrasound Saline infusion sonography Serum estrogen levels

Endometrial sampling The endometrium (lining of the uterus) may develop hyperplasia. This condition is called endometrial hyperplasia. Endometrial hyperplasia is further classified as either simple or complex according to the degree of crowding of the endometrial glands. It is also classified as with atypia (atypical cells) or without atypia based on the appearance of the nucleus of the endometrial cells under a microscope. Risk factors for endometrial hyperplasia include a variety of conditions that increase exposure of the endometrium to unopposed estrogen (estrogen that is not opposed by progesterone), such as taking exogenous unopposed estrogen, obesity, polycystic ovarian syndrome (PCOS), nulliparity, late menopause, and tamoxifen therapy. The risk factors for endometrial hyperplasia are similar to the risk factors for endometrial cancer. The most common clinical presentation of endometrial hyperplasia is abnormal uterine bleeding (AUB) in women with risk factors for endometrial hyperplasia. Abnormal uterine bleeding includes heavy menstrual bleeding, prolonged menstrual bleeding, frequent menstrual bleeding (less than 21 days between menstrual periods), and intermenstrual bleeding. Pelvic examination is often unremarkable in patients with endometrial hyperplasia. The diagnosis of endometrial hyperplasia is based on endometrial sampling, which can be obtained by an in-office endometrial biopsy or during dilation and curettage (D&C) in an operating room. The decision to obtain an endometrial sample from a woman with abnormal uterine bleeding is based on the woman's age, menopausal status, and risk factors for endometrial hyperplasia and cancer. Women younger than 45 years of age with risk factors for endometrial hyperplasia should undergo endometrial sampling. However, women in this age group without risk factors do not require endometrial sampling. These women can initially be managed expectantly or with hormonal contraceptives. Women managed conservatively (without biopsy) who have persistent symptoms may eventually require biopsy. All women who are 45 years of age to menopause with abnormal uterine bleeding require endometrial sampling. All postmenopausal women with any uterine bleeding (including spotting or staining) require further evaluation. Most postmenopausal women are also evaluated with endometrial sampling, but some postmenopausal women may initially be evaluated with pelvic ultrasound to check the thickness of the endometrium. The management of women with endometrial hyperplasia varies based on the type of endometrial hyperplasia (with or without atypia), desire for future fertility, and the presence of risk factors for endometrial cancer. Endometrial hyperplasia without atypical cells has a lower risk of progression to endometrial cancer. Therefore, these women can often be managed with progestin therapy and repeat endometrial sampling. Endometrial hyperplasia with atypical cells has a high risk of concomitant endometrial cancer or progression to endometrial cancer. Women with endometrial hyperplasia with atypia who do not desire future fertility are treated with hysterectomy. Similar women who desire fertility can be treated initially with progestin therapy.

A G2P1 woman who is 31 weeks pregnant with twins presents to the emergency department concerned that she is in labor. She reports a large gush of fluid from her vagina one hour ago. She does not report any contractions. Sterile speculum exam reveals fluid pooling in the vaginal vault, and when the fluid is examined under a microscope, a distinct fern pattern is observed. An ultrasound and fetal heart monitoring are performed and reveal no fetal distress. What is the best management at this time? Administer tocolytics Emergent C-section Expectant management Induce labor

Expectant management This woman presents with preterm prelabor rupture of membranes (PPROM). This presentation is characterized by loss of amniotic fluid prior to initiation of labor (uterine contraction). Smoking, sexually transmitted infections, prior preterm deliveries, and multiple gestations are risk factors for prelabor rupture of membranes (PROM). PROM is diagnosed with a nitrazine paper test, which tests the pH of fluid pooled in the vagina, and a pH > 6.5 is consistent with amniotic fluid. The fern test may also be utilized, which involves microscopically observing the fluid, and a fern pattern is consistent with amniotic fluid. When a patient presents with PROM, it is important to evaluate the status of the fetus with ultrasound and fetal heart monitoring. PROMis defined by rupture of the membranes prior to uterine contractions whereas PPROM is defined by rupture of the membranes between 23 to 37 + 0 weeks of gestation. Management of PROM depends on gestational age of the fetus and whether or not the fetus is in distress. If the fetus is 24-33 weeks gestation, expectant management,group B Streptococcus prophylaxis, and a single dose of corticosteroids are all part of PROM management. If the fetus is between 34-36 6/7 weeks gestation the patient should still prepare for delivery of the fetus, and group B Streptococcus prophylaxis should be given, however, a single dose of corticosteroids is only administered if preterm delivery is expected within seven days.

Which of the following is the most appropriate medication for the treatment of mild to moderate gastroesophageal reflux disease in an otherwise healthy adult? Famotidine Metoclopramide Omeprazole Sucralfate

Famotidine Medical treatment is in a step-up fashion, increasing the potency of treatment until symptoms are controlled. Antacids may be used for symptoms that occur less than once per week. Histamine 2 receptor antagonists such as cimetidine or famotidine are first-line medications for patients with mild to moderatesymptoms. These medications decrease acid secretion by acting on the histamine 2 receptor found on the gastric parietal cells. Patients with symptoms not controlled with histamine 2 receptor antagonists may need treatment with proton pump inhibitors as the next step up in treatment potency. Untreated or poorly treated GERD can lead to complications such as erosive esophagitis, peptic stricture, esophageal ulcers or Barrett esophagus.

A 71-year-old woman presents to the office complaining of "fullness" in her right groin without a visible bulge for the past two weeks. She reports feeling increased "fullness" in the pelvis when she stands and points to an area below the inguinal ligament as her site of pain. On examination, no bulge is palpated in the groin, but the patient is tender to palpation. Pelvic exam is normal. Which of the following is the most likely diagnosis? Abdominal hernia Femoral hernia Inguinal hernia Uterine prolapse

Femoral hernia

Which of the following findings is most suggestive of pyloric stenosis? Five-month-old infant with bloody diarrhea Five-month-old infant with projectile vomiting Five-week-old infant with bloody diarrhea Five-week-old infant with projectile vomiting

Five-week-old infant with projectile vomiting Pyloric stenosis is a disorder of young infants caused by hypertrophy and hyperplasia of the pylorus, which can cause obstruction of the gastric outlet and lead to forceful vomiting. Symptoms usually begin between three and six weeks of age and rarely occur after 12 weeks of age. It is more common in white males and in infants born preterm. The classic presentation is a three- to six-week-old infant who develops immediate postprandial vomiting that is nonbilious and forceful. Due to the forceful nature, it is often called projectile vomiting. These infants also may be referred to as "hungry vomiters" because they want to be refed soon after vomiting. An olive-shaped mass is often palpable in the right upper quadrant. Laboratory evaluation reveals hypochloremic metabolic alkalosis in a minority of cases due to the loss of large amounts of gastric acid. The diagnosis is confirmed with an abdominal ultrasound. Upper GI contrast study is generally used only if the physical examination and ultrasonography are nondiagnostic. The classic sign on UGI is an elongated pyloric canal (the "string" sign). Definitive management of pyloric stenosis is with surgical pyloromyotomy. Other differentials to consider in the infant with vomiting are physiologic gastroesophageal reflux, intestinal obstruction, and adrenal crisis. The immediate postprandial timing and forceful nature of the vomiting in pyloric stenosis help to distinguish it from physiologic gastroesophageal reflux, which usually occurs more than 10 minutes after a meal.

A 22-year-old woman presents to your office to request help with treating her eating disorder. She says that she recently started a counseling program to address bulimia nervosa and her therapist advised starting a medication as part of her treatment. Which of the following is the most appropriate therapy? Bupropion Fluoxetine Nortriptyline Topiramate

Fluoxetine Bupropion (A) is an antidepressant contraindicated for use in patients with bulimia nervosa as it can cause seizures in patients who are actively binging and purging. Nortriptyline (C) is a tricyclic antidepressant that is considered third-line in the treatment of bulimia nervosa due to the side effect profile, which can include weight gain. Nortriptyline is also toxic in overdose, so care should be taken in prescribing this medication if the patient has expressed any suicidal ideation. Topiramate (D) is an anticonvulsant agent that is sometimes used in the treatment of bulimia nervosa when other medications have failed. Side effects include weight loss, which can be problematic for patients with eating disorders and would make its use with normal or underweight patients contraindicated.

A 47-year-old man presents with increased difficulty swallowing and 10 pounds of unintentional weight loss over the last month. He also reports loss of energy. His medical history is notable for prediabetes, gastroesophageal reflux disease, and Helicobacter pylori infection. He is a 20 pack-year smoker who consumes an average of two to three alcoholic beverages daily. Endoscopy with biopsy is significant for an adenocarcinoma at the esophagogastric junction. Which of the following risk factors is most commonly reported in patients with esophageal adenocarcinoma? Alcohol consumption Gastroesophageal reflux disease Helicobacter pylori infection Smoking

Gastroesophageal reflux disease Esophageal cancers are a high-mortality squamous cell carcinoma or adenocarcinoma. The epidemiology, etiology, and risk factors are different for each. Esophageal adenocarcinoma specifically is rising in incidence in industrialized countries. These neoplasms most commonly originate from an area of Barrett metaplasia at the esophagogastric junction, a sequela of gastroesophageal reflux disease. Risk factors for esophageal adenocarcinoma seem to be both independent and synchronous, meaning that they are more strongly associated with the disease in combination than on their own. Notable risk factors on top of gastroesophageal reflux disease include advancing age, smoking, increased body mass index, and a diet low in fruits and vegetables. Early adenocarcinomas in the area of Barrett esophagus are usually asymptomatic. In patients with suspected Barrett esophagus, initial endoscopic biopsy is currently recommended, with subsequent surveillance biopsies depending on the initial findings. With more advanced disease, the most common presenting concerns are progressive dysphagia (initially solids only, then to liquids) and unintentional weight loss. Occult blood loss may also lead to iron deficiency anemia. Diagnosis of esophageal cancer requires an endoscopic biopsy. Treatment and prognosis are directly related to disease stage, both at the local site and any distant metastases.

What is the most common cause of chronic gastritis? Autoimmune Gastrointestinal reflux Helicobacter pylori infection Nonsteroidal anti-inflammatory use

Helicobacter pylori infection

Which of the following studies is the serologic hallmark of an active hepatitis B infection? Hepatitis B core antibody (anti-HBc) Hepatitis B core antigen (HBcAg) Hepatitis B e antigen (HBeAg) Hepatitis B surface antigen (HBsAg)

Hepatitis B surface antigen (HBsAg)

A 30-year-old woman returns to her gynecologist six months after she has delivered a healthy baby. She is currently breastfeeding. She would like to resume oral contraceptive pills as her family planning method. She has a past medical history of hirsutism and deep vein thrombosis. Which of the following is considered a reason to recommend an alternative form of contraception? Current breastfeeding History of deep vein thrombosis History of hirsutism Recent pregnancy

History of deep vein thrombosis Current breastfeeding (A) is not a contradiction for hormonal contraception after four weeks postpartum. There are theoretical concerns that progesterone, specifically, could interfere with milk production. However, most available data do not support this, and both the World Health Organization and the United States Centers for Disease Control have advised that the benefits of progestin-only contraceptives outweigh any theoretical risks. History of hirsutism (C) may be a reason to begin hormonal contraceptives. Estrogen, specifically, acts to increase sex hormone binding globulin (SHBG) production, which then reduces the free androgen that causes male-pattern hair growth characteristic of hirsutism. Recent pregnancy (D) can be a concern for starting contraceptives containing estrogen due to risk of thromboembolism. However, after 30 days postpartum, there is no additional risk for women who do not have other risk factors for thromboembolic events.

A 28-year-old woman presents to her gynecologist for her annual visit including a routine Pap smear. The cytology report notes findings of atypical squamous cells of undetermined significance (ASC-US). Which of the following is the best follow-up test for this finding? Cervical conization Colposcopy Human papillomavirus testing Repeat Pap smear in one year

Human papillomavirus testing The Papanicolaou (or widely known as the Pap) test samples cells from the cervix and vagina to screen forcervical cancer. Current guidelines from the USPSTF and ACOG recommend women should be tested every three years beginning at 21 years of age. Women with an abnormal Pap smear should have follow-up testingappropriate for their age and cytologic findings. Women 21-24 years of age with atypical cytology findings have a different diagnostic algorithm than women > 24 years old. Abnormal findings are classified as (in order from least to most concerning): atypical squamous cells of undetermined significance (ASC-US), atypical squamous cells cannot exclude high-grade squamous intraepithelial lesion (ASC-H), low-grade intraepithelial lesion (LSIL), atypical glandular cells of undetermined significance (AGC), and high-grade squamous intraepithelial lesion (HSIL). The most commonly reported abnormal cytologic finding from Pap smear is atypical squamous cells of undetermined significance (ASC-US). Because human papillomavirus (HPV) DNA are found in nearly all women with cervical cancer, the preferred follow-up test for women > 24 years old whose cytology report showsASC-US is HPV DNA testing. In many clinical settings, combined HPV DNA testing and cervical cancer screening are already standard of care. If the HPV DNA results are negative, then concurrent HPV and cytology are recommended in three years. If the HPV DNA results are positive, colposcopy is the next step.

Which of the following diagnoses is most likely in a gravid woman presenting in her 18th week of pregnancy with hypertension, emesis, proteinuria, and a beta-human chorionic gonadotropin level of > 100,000 mIU/mL? Choriocarcinoma Gestational diabetes Hydatidiform mole Preeclampsia

Hydatidiform mole A hydatidiform mole is a disorder arising from aberrant development of placental trophoblastic tissue. Hydatidiform moles, or molar pregnancies, can be classified as either complete or partial and are part of a larger classification of disorders known as gestational trophoblastic diseases. Complete moles are 46XX and are usually formed from the union of an ovum without chromosomal tissue and one or two sperm that duplicates its genome. Partial moles are triploid (69XXX, 69XXY, or 69XYY) and contain both maternal and paternal genetic material. Partial moles may be associated with fetal development, though the fetus is not viable. While 80% of molar pregnancies are benign, they carry a risk for malignant potential. Complete molar pregnancies carry a higher risk of malignancy with potential for development of choriocarcionoma which can lead to quick metastasis. Risk factors include prior molar pregnancy and extremes in maternal age (younger than 20 years or older than 35 years). Women may present with a positive pregnancy test, painless vaginal bleeding, and pelvic fullness larger than expected for gestational time frame. Elevated levels of beta-human chorionic gonadotropin lead to hyperemesis gravidarum and stimulate growth of bilateral theca lutein cysts, which may be palpated on the ovaries. Signs and symptoms of clinical hyperthyroidism (tachycardia, tremor) may be present due to structural similarities between human chorionic gonadotropin and thyroid-stimulating hormone. On physical exam, the fundal height will be higher than expected for the length of gestation. The physical exam should also include a general assessment of lungs, heart, abdomen, and internal exam of the vagina to exclude the presence of metastasis. The presence of hypertension and proteinuria prior to 20 weeks gestation suggestive of preeclampsia is highly indicative of molar pregnancy. Quantitative serum human chorionic gonadotropin levels should be obtained. Women with significantly elevated levels (> 100,000 mIU/mL) should undergo transvaginal ultrasound, which may reveal a "snowstorm" or "cluster of grapes" pattern without the presence of an embryo or fetus and absent amniotic fluid in a complete molar pregnancy. A growth-restricted fetus may be present on ultrasound of a partial mole. A complete blood count, kidney function tests, liver function tests, thyroid function tests, and a urine protein test are useful tests where concomitant preeclampsia is being considered. Treatment consists of immediate uterine suction curettage followed by weekly human chorionic gonadotropin level measurements until levels return to normal, followed by monthly level measurements for six months to monitor for postmolar gestational trophoblastic neoplasia. Pregnancy is discouraged for up to one year after molar pregnancy to ensure a return to normal human chorionic gonadotropin levels. Gestational trophoblastic neoplasia includes choriocarcinoma, placental site trophoblastic tumor, and invasive mole, all of which can metastasize and necessitate referral to a gynecologic oncologist.

A G1P0 25-year-old woman who is 23 weeks gestation presents to the emergency department with severe abdominal pain and vaginal bleeding. On exam, the uterus is rigid and tender. Fetal heart tones reveal a pulse of 90 bpm. Which of the following is a risk factor for this condition? Gestational diabetes History of sexually transmitted infection Hypertension Incompetent cervix

Hypertension Abruptio placentae is a complication of pregnancy where the placenta prematurely detaches from the uterine wall after 20 weeks gestation and prior to delivery of the infant, however more than half occur at term. Risk factors for abruptio placentae include smoking, cocaine use, trauma, hypertension, and prelabor rupture of membranes. Hypertension is the most common cause of abruptio placentae. Patients will present with severe abdominal pain and dark red blood bleeding from the vagina. On exam, there will be abnormalities in the fetal heart rate, most commonly bradycardia (a normal fetal heart rate is 120-160 bpm). There will also be changes in the mother's hemodynamic status due to blood loss. A pelvic ultrasound aids in the diagnosis of abruptio placentae. Management requires stabilization of the mother's hemodynamic status and immediate delivery. C-section is the preferred method of delivery. Prognosis depends on multiple factors, including gestational age of the fetus and severity of the abruption. Fetal demise and disseminated intravascular coagulation are serious complications of abruptio placentae.

A previously healthy, nulliparous, non-smoking 18-year-old woman presents to your clinic to discuss options for contraception. She plans to attend four-year university next year and does not want a child until well after she finishes school. Which of the following is the most appropriate contraception given her current reproductive plans? Diaphragm Implant Natural family planning Oral contraception

Implant Contraception is used to provide control over the timing of reproduction and to help prevent unwanted or unintended pregnancy. It provides women the opportunity to pursue life goals and contributes to a sense of well-being and self-actualization. The choice of which type of contraception to use depends on a number of factors including efficacy, safety, side effects, access, ease, prevention of sexually transmitted diseases, cost, noncontraceptive benefits, and personal beliefs. Counseling about contraceptive options by the medical provider allows for the patient to be educated about options, dispels misinformation, and helps her select a method of contraception that she will be comfortable using. Women should be counseled to choose the most effective method of contraception based on their reproductive goals, current state of health, and risk factors. Selection of methods starts with determining the patient's reproductive plans and timing of desired pregnancy. Patients who are interested in conception within one year should be given information about short-term, reversible types of contraception, such as the pill, patch, or vaginal ring. Women who seek longer-term contraceptionand who have no plans for pregnancy in the next year should be counseled about long-acting contraception options, including the implant or the intrauterine device (IUD). These types of contraceptives are the most effective at preventing pregnancy and involve simple procedures to place, making them the recommended type of contraception for women wishing to avoid pregnancy in the near future.

A 25-year-old G3P2 woman at eight weeks zero days of gestation presents to the emergency department complaining of pelvic pain and vaginal bleeding. Pelvic examination confirms the uterus as the source of bleeding and reveals a dilated cervix. No products of conception are seen. Pelvic ultrasound shows an embryo with a crown-rump length of 9 millimeters without fetal cardiac activity. Which of the following is the most likely diagnosis? Complete abortion Incomplete abortion Inevitable abortion Threatened abortion

Inevitable abortion

Which of the following best describes the most common cause of an anorectal fistula? Infected anal crypt gland Inflammation secondary to Crohn's disease Obstetric injury Retained rectal foreign body

Infected anal crypt gland Anorectal fistula describes a connection between the anus and the perianal skin. The most common cause of anorectal fistula formation is infection of an anal crypt gland. The rupture of an abscess in the gland leads to formation of an epithelialized tract that communicates with the skin surrounding the anus. Anorectal fistulas are more common in men than women and present as rectal or anal pain with chronic, malodorous drainage from a perianal wound. Simple anal fistulas begin at the dentate line, track through the space between the internal and external sphincters, and exit the skin near the anus. They can be diagnosed on physical exam by noting an opening near the anus and palpating an indurated cord leading to the anus. A soft probe can also be used to follow the tract of the fistula from its termination at the skin to its origin in the anus. Complex anal fistulas involve more than 30% of the external sphincter, originate proximal to the internal sphincter, or originate proximal to the dentate line. Complex anal fistulas can be diagnosed using endosonography, CT scan, MRI, or fistulography. Surgical correction of the fistula tract while preserving fecal continence is the treatment of choice.

Which of the following is the primary risk factor for the development of gastric adenocarcinoma? Consumption of alcohol Infection with Helicobacter pylori Pernicious anemia Smoking

Infection with Helicobacter pylori

Which of the following is a risk factor for the development of ovarian cancer? Infertility Lactation Multiparity Oral contraception

Infertility Infertility is a known risk factor for the development of ovarian cancer. Ovarian cancer is the most common cause of gynecologic cancer death in the United States and is overall the second most common gynecologic malignancy after uterine cancer. Epithelial ovarian carcinoma is the most common histology accounting for roughly 70% of cases. In addition to infertility, other risk factors for ovarian cancer include increased age, family history, early menarche, late menopause, nulliparity, polycystic ovarian syndrome (PCOS), usage of an intrauterine device, and the presence of BRCA1 and BRCA2 gene mutations. Cigarette smoking increases the risk of mucinous ovarian carcinoma. Protective factors include the use of oral contraceptives, multiparity, lactation, tubal ligation, hysterectomy, and salpingo-oophorectomy. Patients often complain of vague abdominal symptoms including bloating, abdominal or pelvic pain, early satiety, and dysuria. Patients can also acutely present with symptoms due to advanced disease such as malignant pleural effusion, bowel obstruction, or venous thromboembolism. Because the symptoms of ovarian cancer are often vague and mildin beginning stages, many women unfortunately evade diagnosis until more advanced stages.The physical exam may reveal abdominal distention due to ascites or a large abdominal mass. Pelvic exam commonly reveals a fixed, irregular adnexal mass. Patients presenting with symptoms and evidence of an adnexal mass on exam should undergo transabdominal and transvaginal ultrasonography as well as pregnancy testing. Diagnosis of ovarian carcinoma is made from pathological examination of ovarian tissue obtained from surgical exploration or image-guided biopsy of the lesion. A computed tomography (CT) scan of the abdomen and pelvis is useful in assessing for peritoneal metastasis as well as lymph node involvement. Chest radiograph or CT is useful in patients when malignant pleural effusion or metastasis to mediastinal lymph nodes is suspected. Serum cancer antigen 125 (CA 125) is a known tumor marker for ovarian cancerthat is also elevated in a number of other disease states. It is used to monitor response to therapy over time. Treatment of ovarian cancer consists of surgical excision and debulking of tumor burden, and chemotherapy. The five-year survival rate for stage III ovarian cancer is < 50% and < 20% for stage IV. Patients are seen every few months for regular exams and CA 125 measurements for several years after the conclusion of treatment.

A 42-year-old man presents with a protrusion in the right groin that has been progressing over the past two months. The bulge worsens with straining and reduces when he lies flat. He denies any pain or changes in bowel habits. On exam, you palpate a bulge when your finger is adjacent to the spermatic cord during valsalva maneuver. Which of the following types of hernia is highest on your differential diagnosis list? Abdominal Femoral Incisional Inguinal

Inguinal Inguinal hernia is described in this vignette. A hernia is defined as a protrusion, bulge, or projection of an organ or part of an organ through the body wall that normally contains it. Both inguinal and femoral hernias are collectively considered groin hernias. Of the groin hernias, inguinal hernias are more common than femoral hernias and consist of either direct or indirect inguinal hernia. Hernias are more common in men than women, and when present in women, they develop at a later age. Risk factors for development of a hernia are previous hernia repair, family history of hernia, abdominal wall injury, Caucasian race, male sex, chronic constipation, chronic cough, and smoking. Indirect inguinal hernias are the most common type of hernia requiring repair. They occur when the protrusion is into the internal inguinal ring, lateral to the inferior epigastric artery into the spermatic cord in men, and the round ligament in women. These are often due to a congenital deformity in which there is persistent patent processus vaginalis and failure of the inguinal ring to completely close. Although indirect inguinal hernias are due to congenital deformity, they are often not recognized in the neonatal period or during childhood, but rather in adulthood when reduced muscle tone causes widening of the internal inguinal ring, subsequently leading to a detectable hernia. A direct inguinal hernia is characterized by a protrusion into Hesselbach's triangle, which is medial to the inferior epigastric artery. Direct hernias are typically acquired later in life from chronic overstretching or injury resulting in laxity of the abdominal musculature. Femoral hernias protrude inferior to the inguinal ligament and into the femoral ring. Femoral hernias are most commonly seen in women and often present with signs of incarceration and strangulation warranting emergency treatment. Clinical presentation varies based on the severity and type of hernia. The patient may be asymptomatic or may report swelling or fullness at the hernia site which may enlarge with increased intra-abdominal pressure and standing. In more severe cases, the hernia may become incarcerated, in which there is a painful enlargement of hernia that is not able to be reduced through the defect. This will present with symptoms of bowel obstruction such as obstipation, vomiting, painful defecation, and severe tenderness. Prolonged hernia incarceration may lead to ischemia and necrosis of the hernia, termed strangulation, leading to systemic toxicity and severe pain. Diagnosis is initially made based on history and physical examination, however, a groin ultrasound is the best imaging modality when the diagnosis is not readily apparent, or the type of hernia cannot be determined based on clinical examination alone. Abdominal CT should be ordered in patients with manifestations of bowel obstruction who do not warrant immediate surgical treatment. All complicated hernias, inguinal or femoral, require surgical repair, either open or laparoscopic to prevent progression. Once a hernia becomes strangulated, meaning a loop of bowel is caught in the hernia and is unable to allow passage of fecal matter, it is considered a surgical emergency to prevent ischemia and necrosis of the involved bowel loop. Therefore, when identified, an early an attempt to prevent progression to this stage should be made. A non-complicated hernia surgical repair is meant to improve patient symptoms and prevent progression. However, if a hernia is asymptomatic and not complicated by bowel protruding through the hernia, then watchful waiting is an appropriate option.

A 19-year-old woman presents to the office for her annual wellness exam and is found to have a body mass index of 17.1 kg/m2. When asked what she thinks of her weight, she states that she has weight that she needs to lose. You question further and suspect anorexia nervosa. She is medically stable, and there is no other explanation for her weight loss. Which of the following is the best treatment plan for this patient? Cognitive-behavioral therapy Interdisciplinary approach Nutritional rehabilitation Selective-serotonin reuptake inhibitors

Interdisciplinary approach Cognitive-behavioral therapy (A) is a common form of psychotherapy that is helpful in the management of anorexia nervosa. It is not, however, sufficient to be used alone in management of this disorder. Nutritional rehabilitation (C) is needed to restore the patient to healthy body weight. It is not, however, sufficient as stand-alone therapy. Patients often return to their restrictive behavior once their weight is healthy, because they have not addressed their disordered relationship with food and weight. Selective-serotonin reuptake inhibitors (D) can be used as adjuvant treatment for anorexia nervosa but are not indicated as monotherapy.

Which of the following clinical findings is most suggestive of a small bowel obstruction? Constant left lower quadrant abdominal pain Intermittent crampy abdominal pain with abdominal distention Loose nonbloody stools Right upper quadrant tenderness with deep inspiration

Intermittent crampy abdominal pain with abdominal distention Small bowel obstruction (SBO) occurs when the normal flow of intraluminal contents in the small bowel is interrupted. Obstruction is often due to a mechanical obstruction and can be partial or complete. The most common causes of mechanical small bowel obstruction are adhesions, hernias, and tumors. Adhesions are usually due to prior intra-abdominal surgeries causing postoperative adhesions but can also occur due to inflammatory conditions such as Crohn's disease or diverticular disease. Small bowel obstruction leads to bowel dilation proximal to the obstruction, which causes abdominal distention. The clinical presentation of mechanical small bowel obstruction depends upon the site and etiology of obstruction. The classic presentation is intermittent cramping abdominal pain, nausea, vomiting, and abdominal distention. The vomiting may be bilious. Obstipation, which is the inability to pass gas or stool, is a common finding as well. Proximal obstructions generally cause more frequent vomiting and distal obstructions cause more abdominal distention. Another hallmark finding is dehydration, which presents as hypotension, tachycardia, and oliguria. Laboratory findings can be helpful in assessing severity and complications. Elevated serum lactate is sensitive, but not specific, for bowel ischemia. The diagnosis is suggested based on risk factors and clinical findings but confirmed with imaging. Plain abdominal radiographs may reveal dilated loops of bowel proximal to the obstruction and air-fluid levels. Abdominal computed tomography scans are more accurate for identifying the site, severity (partial or complete), and cause of the obstruction. Patients with partial small bowel obstructions without complications can initially be managed nonoperatively. Medical management consists of intravenous fluids, correction of electrolyte abnormalities, dietary restriction to nothing by mouth (NPO), gastrointestinal decompression with nasogastric tube, and pain management. Surgical management is indicated when symptoms persist despite medical management or complications are suspected. Surgical management depends on the underlying cause but may include adhesiolysis, hernia repair, or small bowel resection with end-to-end anastomosis.

A 21-year-old woman presents to the clinic complaining of increased abdominal girth, weight loss, decreased appetite, irritable mood, problems with speech articulation, and a mild tremor. She also complains of generalized pruritus and fatigue. Her father and sister both have Wilson disease, but she has never been tested for it. Which of the following physical exam findings would be most specific for a diagnosis of Wilson disease in this patient? Caput medusae Jaundice Kayser-Fleischer rings Spider angioma

Kayser-Fleischer rings

A premenopausal woman was evaluated for abnormal uterine bleeding and diagnosed with endometrial hyperplasia without atypical cells. She desires children in the future. Which of the following is the preferred first-line treatment? Combined hormonal oral contraceptive Hysterectomy Leuprolide Levonorgestrel-releasing intrauterine device

Levonorgestrel-releasing intrauterine device Combined hormonal oral contraceptive (A) medications are not as well studied for endometrial hyperplasia treatment as progestin-only hormonal therapy. In theory, the amount of progestin in a combined contraceptive is still sufficient; however, due to the lack of studies, the levonorgestrel-releasing intrauterine device is first-line. Hysterectomy (B) is the recommended treatment for women with endometrial hyperplasia with atypical cells who do not desire future fertility. Hysterectomy permanently eliminates the opportunity for future fertility. Leuprolide (C)is a gonadotropin-releasing hormone agonist that can be used to treat endometriosis or used preoperatively to reduce the size of fibroids. It has no role in the treatment of endometrial hyperplasia.

Which of the following clinical findings is most common in pelvic inflammatory disease? Abnormal vaginal bleeding Fever Lower abdominal pain Vaginal discharge

Lower abdominal pain Abnormal vaginal bleeding (A) is a less frequent symptom than pelvic pain in patients with pelvic inflammatory disease. Abnormal vaginal bleeding includes menorrhagia, intermenstrual bleeding, and postcoital bleeding. Fever (B) occurs in some patients with pelvic inflammatory disease but is less common than pelvic pain. Fever may suggest a more severe case of pelvic inflammatory disease. Vaginal discharge (D) is also incorrect because it occurs less frequently than pelvic pain.

A 25-year-old woman presents to the office with a history of a painless ulcer on the left labia majora 4 weeks ago, which has since resolved. Physical exam reveals painful inguinal lymph nodes that have coalesced to form buboes, along with rectal pain and bloody purulent discharge. Which of the following is the most likely diagnosis? Chancroid Herpes simplex Lymphogranuloma venereum Syphilis

Lymphogranuloma venereum Lymphogranuloma venereum is an uncommon sexually transmitted disease caused by Chlamydia trachomatistypes L1-L3. It is endemic to parts of Africa, Southeast Asia, India, the Caribbean, and South America. It is rare in industrialized countries, but there has been a rise in cases over the last 10 years in North America and Europe in men who have sex with men. Lymphogranuloma is acquired through sexual intercourse and has an incubation period of 5 to 21 days before symptoms begin. The initial lesion appears as a painless vesicle orulcer, which may go unnoticed. The lesion resolves spontaneously as the infection spreads to the lymph nodes of the genital and rectal area. Within 1 to 4 weeks postexposure, inguinal buboes develop bilaterally and eventually fuse and break down to form multiple draining sinuses. Early in the disease process, patients experience proctitis with tenesmus, or bloody purulent drainage from the anal region. As the disease progresses, patients present with scarring inflammation of the perirectal tissue, along with obstipation, rectal stricture, and rectovaginal and perianal fistulas. Diagnosis is made clinically and can be supported with laboratory findings. The complement fixation test may be positive with high titers, indicating active disease, but can have a cross-reaction with other chlamydial infections. Nucleic acid detection tests are sensitive but are not yet approved by the FDA for this condition. Care must be taken to differentiate lymphogranuloma venereum from other sexually transmitted infections with similar early manifestations, such as syphilis, herpes, and chancroid. Treatment is with doxycycline 100 mg twice per day for 21 days. Incision and drainage of inguinal buboes may be performed in addition to surgical repair in severe cases.

A 24-year-old woman presents to the emergency department complaining of several days of dull abdominal pain that occurs after eating. The episodes last about 10-15 minutes and are associated with nausea. A bedside abdominal ultrasound is ordered and reveals a dilated common bile duct. Which of the following is the least invasive test for detecting common bile duct stones? Endoscopic retrograde cholangiopancreatography Magnetic resonance imaging with magnetic resonance cholangiopancreatography Percutaneous transhepatic cholangiography Plain abdominal radiograph

Magnetic resonance imaging with magnetic resonance cholangiopancreatography Magnetic resonance imaging (MRI) with magnetic resonance cholangiopancreatography (MRCP) is excellent at identifying stones in the biliary tract and is usually reserved for suspected choledocholithiasis. It is often recommended as the second-line imaging study of choice if ultrasonography does not provide a clear diagnosis. Endoscopic retrograde cholangiopancreatography (ERCP) and percutaneous transhepatic cholangiography are diagnostic as well as therapeutic, but both are invasive, require contrast, and have a high risk of complications.

A 28-year-old G1P1 at 41 weeks gestation has a normal spontaneous vaginal delivery. Due to shoulder dystocia during delivery, an episiotomy was performed. Where would the expected episiotomy incision be found? Anteriorly towards the urethral meatus Curved towards the bulbocavernosus muscle Mediolaterally towards the ischial tuberosity Midline extending into the anal sphincter

Mediolaterally towards the ischial tuberosity Mediolaterally towards the ischial tuberosity would be the expected location of a mediolateral episiotomy incision. Episiotomy, now no longer recommended, was previously done to enlarge the vaginal opening and assist in the delivery of the fetus. The procedure is now typically only performed if the patient is at high risk of third- and fourth-degree lacerations. Other indications include poor fetal heart tracing during a vaginal delivery or need for operative vaginal delivery (use of forceps or vacuum extractor). Episiotomy can be performed either midline or mediolaterally. Mediolateral incision is the preferred approach if an episiotomy must be performed. Less commonly performed incisions include J incision, T episiotomy, lateral episiotomy, and anterior episiotomy. Midline incisions have higher risk of extending to the anal sphincter causing more complicated repairs. Other complications of the procedure include poor cosmetic repair, fistula development, increased blood loss, and infection.

Which of the following is a risk factor for the development of pilonidal disease? Age greater than 45 years Female sex Sedentary lifestyle Underweight

Sedentary lifestyle

Which of the following is true regarding the treatment plan of placental abruption? All patients with placental abruption should be delivered at a gestational age of 36 weeks All patients with placental abruption should undergo cesarean delivery All patients with placental abruption should undergo immediate delivery The treatment plan depends on fetal status, maternal hemodynamic stability, and gestational age of the fetus

The treatment plan depends on fetal status, maternal hemodynamic stability, and gestational age of the fetus Placental abruption (abruptio placenta) refers to partial or complete separation of the placenta prior to thedelivery of the fetus. The severity of placental abruption varies based on the acuity of separation and the percentage of the placenta that has been separated. Placental abruption occurs due to ruptured maternal blood vessels beneath the placenta that create a hemorrhage. This hemorrhage separates the placenta from the decidua (endometrium of pregnancy). Most placental abruptions are due to chronic pathologic vascular processes, but some abruptions are secondary to trauma or vasoconstriction. The most important risk factoris a prior placental abruption. Cigarette smoking, cocaine use, hypertension, and multiparity are other important risk factors for placental abruption. The classic clinical manifestations of acute abruption include a sudden onset of abdominal or back pain, vaginal bleeding, and uterine contractions that occur after 20 weeks of gestation. On physical exam, the uterus is often firm and tender. The severity of the pain and bleeding vary, and the amount of visible vaginal bleeding is not predictive of maternal blood loss. Fetal heart rate abnormalities indicate severe placental separation and increase the likelihood of fetal demise. Placental abruption is a clinical diagnosis that is sometimes supported by ultrasound findings. However, only 25-50% of placental abruptions are visible on ultrasound. The management of placental abruption depends on the fetal status, hemodynamic stability of the mother, and fetal heart rate and gestational age. If the fetus is nonviable, then the delivery method depends on the hemodynamic status of the mother. If the mother is hemodynamically stable, then vaginal delivery is preferred. If the mother is hemodynamically unstable, thencesarean delivery is indicated to quickly stop the bleeding. In addition, measures to stabilize a mother who is hemodynamically unstable include intravenous fluids and transfusion of blood products, such as packed red blood cells. Prior to blood transfusion, type and screen and crossmatching must be performed. If the fetus is alive with nonreassuring (category III) fetal heart rate, then delivery is also urgent. If the fetus is alive with reassuring fetal heart rate, then the management depends on the maternal hemodynamic status. If the mother is hemodynamically unstable, then delivery should be done immediately. Vaginal delivery is preferred if delivery is imminent. Placental abruption cases with a stable mother and fetus are managedaccording to gestational age and the progression of symptoms. Conservative management may be used in pregnancies of less than 34 weeks of gestation if both maternal and fetal status are stable. Conservative management includes antenatal corticosteroids and monitoring for disease progression. All cases of acuteplacental abruption presenting at beyond 36 weeks of gestation should be delivered. Vaginal delivery is preferred if there is no obstetric indication for cesarean delivery.

A 71-year-old woman with compensated cirrhosis presents to the gastrointestinal specialty clinic. She has undergone upper endoscopy and is found to have small esophageal varices. What is the first step in pharmacologic management of her varices to prevent advancement and hemorrhage? Calcium channel blockers Nitrates Nonselective beta-blocker Selective beta-blockers

Nonselective beta-blocker Esophageal varices are common in about half of patients with cirrhosis (most common cause in adults) and create a risk of morbidity and mortality due to hemorrhage. The risk of hemorrhage depends on the size of the varices and extent of hepatic dysfunction. Cirrhosis causes vasoconstriction within the liver resulting in increased intravascular pressure in the portal vein system as well as in the collateral veins that also help drain systemic circulation. Varices develop to relieve some of the pressure within the portal vein system. Patients with known cirrhosis should undergo endoscopic screening for varices, and those with increased risk of hemorrhage should be started on prophylaxis treatment to prevent progression and hemorrhage. Pre-primary prophylaxis treatment aims to prevent the development of esophageal varices in patients with cirrhosis that have known portal hypertension. This is achieved with treatment of the cause of liver disease. Primary prophylaxis aims to prevent hemorrhage of known varices with nonselective beta-blockers to lower portal pressures by decreasing hepatic inflow. Common nonselective beta-blockers, such as propranolol and nadolol, work to dilate mesenteric arterioles and therefore decrease hepatic inflow. These medications may not be safe in patients with refractory ascites or bacterial peritonitis. Patients who develop an acute upper gastrointestinal bleed from esophageal varices may present with hematemesis, hematochezia, melena and symptoms of anemia such as fatigue, weakness and feeling faint. Bleeding is often rapid and will require prompt management. The first step is hemodynamic stabilization with fluid resuscitation, use of a vasoactive medication such as terlipressin, somatostatin, or octreotide for three to five days in addition to endoscopic intervention. Endoscopic intervention should be completed within 12 hours of presentation, it may include banding, ligation, or balloon placement to rapidly stop the bleeding. Prior to a procedure like transjugular intrahepatic portosystemic shunt (TIPS) placement for decompression, a repeat endoscopic evaluation and treatment should be performed.

A 71-year-old woman with compensated cirrhosis presents to the gastrointestinal specialty clinic. She has undergone upper endoscopy and is found to have small esophageal varices. What is the first step in pharmacologic management of her varices to prevent advancement and hemorrhage? Calcium channel blockers Nitrates Nonselective beta-blockers Selective beta-blockers

Nonselective beta-blockers Pre-primary prophylaxis treatment aims to prevent the development of esophageal varices in patients with cirrhosis that have known portal hypertension. This is achieved with treatment of the cause of liver disease. Primary prophylaxis aims to prevent hemorrhage of known varices with nonselective beta-blockers to lower portal pressures by decreasing hepatic inflow. Common nonselective beta-blockers, such as propranolol and nadolol, work to dilate mesenteric arterioles and therefore decrease hepatic inflow. These medications may not be safe in patients with refractory ascites or bacterial peritonitis. Patients who develop an acute upper gastrointestinal bleed from esophageal varices may present with hematemesis, hematochezia, melena and symptoms of anemia such as fatigue, weakness and feeling faint. Bleeding is often rapid and will require prompt management. The first step is hemodynamic stabilization with fluid resuscitation, use of a vasoactive medication such as terlipressin, somatostatin, or octreotide for three to five days in addition to endoscopic intervention. Endoscopic intervention should be completed within 12 hours of presentation, it may include banding, ligation, or balloon placement to rapidly stop the bleeding. Prior to a procedure like transjugular intrahepatic portosystemic shunt (TIPS) placement for decompression, a repeat endoscopic evaluation and treatment should be performed.

Which of the following physical examination findings is most concerning for breast cancer? Bilateral, tender, round masses Multiple tender, round masses that fluctuate in size Nontender, firm, immobile mass Round, firm, mobile, nontender mass

Nontender, firm, immobile mass

Which of the following is the most useful initial laboratory test in a patient presenting with secondary amenorrhea? Follicle-stimulating hormone Glycated hemoglobin Prolactin Serum beta-human chorionic gonadotropin

Serum beta-human chorionic gonadotropin The most important first step in the diagnostic workup of both primary and secondary amenorrhea is to rule out pregnancy, which can be done with either a serum beta-human chorionic gonadotropin (hCG) measurement or urine beta-hCG measurement.

What is the primary cause of acute appendicitis? Adhesions Bacterial overgrowth Ileocecal valve dysfunction Obstruction of the appendix

Obstruction of the appendix Obstruction of the appendix is believed to be the primary cause of appendicitis. This source is sometimes identified during laparoscopy but not always. Obstruction may be due to fecaliths (hard fecal masses), calculi, lymphoid hyperplasia, infectious processes, and benign or malignant tumors. Obstruction of the appendix leads to an increase in intraluminal pressure, which then induces thrombosis and occlusion of the small vessels in the appendiceal wall and stasis of lymphatic flow. Stasis results in the appendix becoming engorged.

A 21-year-old woman presents with irregular menses, hirsutism, and acne of six months duration. She currently does not wish to become pregnant. Which of the following is the first-line pharmacologic treatment for this condition? Clomiphene citrate Metformin Oral contraceptive pills Spironolactone

Oral contraceptive pills Oral contraceptive pills are the first-line pharmacologic treatment for polycystic ovarian syndrome, which is described in this vignette. Polycystic ovarian syndrome is a common endocrine disorder affecting 5-10% of women of reproductive age and is characterized by the triad of increased androgen production, ovulatory dysfunction,and polycystic ovaries. The condition is associated with hirsutism, obesity, increased risk of diabetes,cardiovascular disease, and metabolic syndrome. Patients present with menstrual dysfunction, ranging fromamenorrhea to menorrhagia, and often experience infertility. Due to androgen excess, women may experience acne and hirsutism. Diagnosis is made clinically by identifying at least two out of the three criteria (using Rotterdam criteria). Other conditions causing hyperandrogenism and menstrual irregularity must be excluded, such as thyroid dysfunction, adrenal and ovarian tumors, congenital adrenal hyperplasia, Cushing syndrome, hyperprolactinemia, and acromegaly. Transvaginal ultrasound is useful in identifying polycystic ovaries. Screening lab work should be ordered to exclude other diagnoses and to support the diagnosis. Androgen excess of polycystic ovarian syndrome is frequently identified by elevated free testosterone and normal or slightly elevated DHEA-S levels. The follicle-stimulating hormone (FSH) is usually normal or slightly low, with luteinizing hormone (LH) elevated according to Tanner stage, sex, and age. The LH-to-FSH ratio is usually greater than three. Treatment is initially conservative with lifestyle modifications, such as diet and exercise, to stimulate ovulation and reverse metabolic effects. Medical treatment should be reserved for controlling metabolic conditions and regulating menses. Oral contraceptive pills should be employed first to regulate menstruation and inhibit ovarian androgen production. If androgen production is not adequately inhibited, an androgen blocking agent, such as spironolactone, may be employed to reduce the symptoms of hirsutism and acne. When fertility is desired, first-line treatment of ovulatory induction is with clomiphene citrate. Metformin is beneficial for women experiencing metabolic and glucose abnormalities and can help prevent the development of diabetes. While metformin is very useful in controlling metabolic abnormalities, it has no effect on hyperandrogenism or infertility.

A 25-year-old woman is diagnosed with a ruptured ovarian cyst. She is hemodynamically stable, and a pelvic ultrasound shows a small amount of peritoneal fluid. Which of the following treatment plans is most appropriate? Hospitalization with close observation Outpatient management with oral analgesia Outpatient management with oral antibiotics Surgical intervention

Outpatient management with oral analgesia Hospitalization with close observation (A) is the management for patients with a ruptured ovarian cyst and a large or enlarging hemorrhage. These patients require hospitalization for close monitoring and possible surgery. Outpatient management with oral antibiotics (C) is the management for uncomplicated infections, such as cystitis, sexually transmitted infections, and some patients with pelvic inflammatory disease. Surgical intervention (D) is indicated for patients with a ruptured ovarian cyst who are hemodynamically unstable or who have an enlarging hemorrhage.

A healthy 23-year-old woman presents to your office for her annual exam. She has never undergone screening for cervical cancer and says that she has never been sexually active. Which of the following is the most appropriate next step according to the USPSTF? No screening needed until sexually active Papanicolaou plus human papillomavirus testing Papanicolaou test alone Pelvic ultrasound

Papanicolaou test alone Screening is based on the patient's age and previous PAP results and starts at age 21 years. Recommended screening for women aged 21-29 yearsis with the PAP test alone every three years when the cytology results are normal. Follow-up to abnormal results depends on the cytology findings and can range from a repeat PAP in one year to colposcopy. Women aged 21-29 years are more likely to have transient HPV infections that clear spontaneously without any intervention. HPV testing in this group can lead to unnecessary and uncomfortable interventions that can cause a lot of stress and cost for the patient, therefore, routine HPV testing is not recommended until after age 29 years.

Infection with Helicobacter pylori can lead to the development of what condition? Acute cholangitis Diarrhea Peptic ulcer disease Pneumonia

Peptic ulcer disease

Which of the following physical exam findings is most indicative of acute peritonitis? Fluid wave Hyperresonance Periumbilical ecchymosis Peritonitis

Peritonitis Fluid wave (A) describes a physical exam maneuver in which the patient places their hand on the midline of the abdomen while the examiner taps one side of the abdomen and places their opposite hand on the other side of the abdomen. The test is considered positive if the tap is felt in the receiving hand. The fluid wave maneuver tests for free fluid in the abdomen, as is found in ascites. While patients with ascites can suffer from acute peritonitis, the fluid wave test is not indicative of peritoneal irritation. Hyperresonance (B) of the abdomen can be appreciated by placing two fingers on the abdomen and tapping them with the opposite fingers. A high pitched, drum-like sound indicates areas of air accumulation in the abdomen (usually in the intestines). A dull, low-pitched sound indicates solid structures, such as organs, tumors, or stool. Hyperresonance can indicate increased intestinal gas or free air in the abdominal cavity, as can happen in a perforation. Thus, while hyperresonance may be observed in patients with peritonitis, it is not a specific finding. Periumbilical ecchymosis (C), or Cullen's sign, describes bruising around or just inferior to the umbilicus. Disease states that can cause periumbilical ecchymosis include aortic rupture, acute pancreatitis, and ectopic pregnancy. Many patients with periumbilical ecchymosis will have concurrent peritonitis, but it is not considered a specific sign of peritoneal irritation and is not a common finding.

Which of the following systemic diseases is associated with an increased risk for formation of colonic polyps? Multiple endocrine neoplasia type I Neurofibromatosis type I Peutz-Jeghers syndrome Systemic lupus erythematosus

Peutz-Jeghers syndrome Colonic polyps are growths that protrude into the lumen of the colon and can be inflammatory, hamartomatous, serrated, or adenomatous. Inflammatory colonic polyps are nonneoplastic, but each of the other categories has the potential for dysplasia and malignancy. Risk factors for the development of inflammatory polyps include inflammatory bowel disease, rectal prolapse, or diverticular disease. While inflammatory polyps do not generally become malignant, the tissue surrounding them can be of malignant concern in patients with chronic bowel inflammation. Hamartomatous polyps are disorganized masses of typical colonic cells. The polyps of Peutz-Jeghers syndrome are hamartomatous and made up of accumulations of smooth muscle cells and glandular epithelium. Peutz-Jeghers syndrome is a rare genetic disorder that leads to the development of multiple colonic polyps. These polyps have the potential to become neoplastic, necessitating frequent colonoscopy and close surveillance. Other risk factors for the development of hamartomatous polyps include several genetically-acquired syndromes such as Cowden syndrome and juvenile polyposis syndrome. Serrated polyps have a sawtooth appearance, are not often associated with genetic syndromes, and are infrequently neoplastic. The risk of developing dysplasia in a serrated polyp appears to be related to size and location. Lesions that are larger or more proximally located are considered to be at increased risk for malignant development. Adenomatous polypsare the most common type of colonic polyp. Risk factors for their development include age (greater risk as age increases over 50 years), obesity, male gender, and African American ethnicity, although adenomatous polyps can be found in anyone at almost any age. Several genetic disorders can also cause adenomatous polyps to occur in a hereditary pattern. Two such syndromes include familial adenomatous polyposis and Lynch syndrome. Symptoms of colonic polyps include bloody stools, constipation, and abdominal pain, although most polyps are asymptomatic. Adults of average risk should be screened for colon polyps starting at age 45 years, and those at increased risk for colonic polyps should begin screening by age 40 years. Screening can include any of the following modalities: colonoscopy every 10 years, flexible sigmoidoscopy every five years, computed tomographic colonography every five years, or stool-based tests yearly. A positive screening test should be followed-up by colonoscopy. Treatment for colon polyps with malignant potential is with removal during colonoscopy and more frequent screening afterward.

Which of the following diagnostic studies best confirms the diagnosis of genital herpes simplex virus in a patient with active lesions? Polymerase chain reaction-based testing Serologic testing Tzanck smear Wet mount microscopy technique

Polymerase chain reaction-based testing Genital herpes simplex virus (HSV) is a common sexually transmitted disease. It has two subtypes: herpes simplex virus type 1 (HSV-1) and herpes simplex virus type 2 (HSV-2). Either subtype can cause genital or orolabial symptoms, despite the classic association of herpes simplex virus type 2 causing genital symptoms and herpes simplex virus type 1 causing orolabial symptoms. Transmission of genital herpes simplex virus is more likely to occur during active lesions, but it can also occur during asymptomatic periods. The initial infection of genital herpes simplex virus is referred to as the primary infection. Primary herpes simplex virus infection classically presents with multiple vesicular lesions on an erythematous base and systemic symptoms, such as fever, malaise, chills, and myalgias. Tender inguinal lymphadenopathy is also common. However, there is wide variability in the severity of symptoms caused by herpes simplex virus infection. Patients with herpes simplex virus infection frequently have recurrent symptomatic episodes. These episodes occur because herpes simplex virus is never eradicated, but rather, it remains in a latent state in the nerve roots between episodes. Recurrent episodes occur during reactivation of the virus and typically have similar, but less severe, symptoms and a shorter symptom duration. The frequency of recurrent episodes varies widely between patients. Some patients have prodromal symptoms, such as local tingling or shooting pain, prior to the recurrent vesicular lesions. Herpes simplex virus infection should be confirmed with a diagnostic study because the clinical findings are nonspecific. Viral culture or a polymerase chain reaction-based study is the preferred diagnostic study in patients who have active lesions. Serologic testing can be used to confirm a patient has been exposed to herpes simplex virus in the past, but it does not differentiate prior herpes simplex virus infection from current herpes simplex virus infection and therefore cannot be used alone for diagnosis. Antiviral therapy from the nucleoside-analog class is used to treat herpes simplex virus infections. Examples of nucleoside analogs include acyclovir, valacyclovir, and famciclovir. Primary infections should always be treated with antiviral therapy and are treated for a longer duration than recurrent infections. There are multiple approaches to the treatment of recurrent herpes simplex virus episodes. Recurrent episodes can be managed without treatment, withepisodic treatment, or with chronic suppressive therapy. Patients who have six or more episodes per year, severe symptoms, or severe psychological distress with recurrent symptoms should be managed with chronic suppressive treatment. Patients who have minimal symptoms and infrequent episodes can be managed without treatment.

A 40-year-old woman who is at 22 weeks gestation returns to the office for a routine obstetric appointment. She has a history of benign essential hypertension that is normally controlled with diet and exercise. She has a blood pressure of 154/89 mm Hg with mild proteinuria on dipstick. She has an unremarkable complete blood count. She does not complain of any symptoms, and a physical exam and fetal ultrasound are unremarkable. Which of the following is the most likely diagnosis? Eclampsia Hemolysis, elevated liver enzymes, and low platelet count syndrome Malignant hypertension Preeclampsia

Preeclampsia Preeclampsia is a disorder in pregnant women characterized by elevated blood pressure and proteinuria that occurs after 20 weeks gestation. Requirements for diagnosis include a systolic blood pressure greater than or equal to 140 mm Hg or a diastolic blood pressure greater than or equal to 90 mm Hg on two occasions at least 4 hours apart in a patient who usually has normal blood pressure or a systolic blood pressure greater than or equal to 160 mm Hg or a diastolic blood pressure greater than equal to 110 mm Hg or higher at any given time. The patient must have proteinuria (greater than or equal to 300 mg per 24-hour urine collection or protein to creatinine ratio of greater than 0.3 mg/dL) in addition to blood pressure elevation to be diagnosed with this condition. Patients with preeclampsia can be asymptomatic, but the condition can progress in some patients to affect multiple organs, causing hepatic dysfunction (causing severe persistent upper quadrant or epigastric pain), renal insufficiency, visual disturbances, pulmonary edema, and thrombocytopenia. Patients who have end-organ effects may complain of headache, visual disturbances, altered mental status, blindness, dyspnea, sudden edema, epigastric or right upper quadrant pain, weakness or malaise, and abnormal muscle movements. Eclampsia is diagnosed in a patient who develops seizures in addition to the above signs and symptoms. A complete blood count, liver function test, serum creatinine, uric acid level, and a 24-hour urine collection or urine dipstick analysis should be ordered in any patient suspected to have preeclampsia. A computed tomography scan of the head can rule out intracranial hemorrhage in patients with severe headaches or focal neurologic deficits. Delivery is the only cure for preeclampsia. Benefits and risks of premature birth must be weighed against effects of preeclampsia. Women with preeclampsia without evidence of end-organ damage are usually induced after 37 weeks gestation, while women with end-organ damage are induced after 34 weeks. Hydralazine, labetalol, and nifedipine are some of the drugs used for blood pressure control. First-line treatment of seizures is magnesium sulfate. Symptoms should resolve by 6-12 weeks postpartum.

Which of the following is the most common cause of secondary amenorrhea? Asherman syndrome Endometrial polyps Pregnancy Sheehan syndrome

Pregnancy Pregnancy is the most common cause of secondary amenorrhea. Primary amenorrhea is the absence of menarche by age 15 or thereafter. Secondary amenorrhea is the absence of menses for more than three months in a previously menstruating woman or the absence of menses for more than six months in women with previously irregular menses. Oligomenorrhea is defined as a menstrual cycle length of greater than 35 days or fewer than nine menstrual cycles per year. The most common causes of primary amenorrhea are largely due to genetic anomalies such as gonadal dysgenesis due to Turner syndrome or anatomical anomalies such as Müllerian agenesis. A congenital deficiency of GnRH (Kallmann's syndrome) also presents with primary amenorrhea.The etiology of secondary amenorrhea and oligomenorrhea is similar, and the diagnostic approach is the same. When evaluating causes of secondary amenorrhea, pregnancy must be ruled out first. Other causes of secondary amenorrhea include endocrinopathies such as thyroid disease, Cushing syndrome, and dysfunction anywhere along the hypothalamic-pituitary-ovarian axis. Hypothalamic etiologies include abnormalities of gonadotropin-releasing hormone (GnRH) transport or production, including trauma or infiltrative disease affecting the hypothalamus, sarcoidosis, or systemic illness. Low body mass index (BMI) associated with eating disorders or the female athlete triad can cause a functional hypothalamic amenorrhea. Pituitary etiologies include adenomas, iatrogenic injury, Sheehan syndrome, ormedications, such as antipsychotics. Ovarian causes include premature ovarian failure and polycystic ovarian syndrome (PCOS). Intrauterine adhesions associated with Asherman syndrome can also cause a secondary amenorrhea or oligomenorrhea.

A 40-year-old man presents to his primary care physician for a routine physical exam. He has no significant past medical history, no recent illnesses, and no presenting complaints. His only concerning finding on exam and workup is an elevated serum alkaline phosphatase. Additional blood tests are sent and imaging, including a cholangiogram, is performed. Pertinent results include normal aminotransferase levels, normal serum IgG and IgG4, and normal IgG and cholangiogram notable for areas of intrahepatic and extrahepatic medium and large biliary duct dilation and strictures. Which of the following is the most likely diagnosis? IgG4-associated cholangitis Primary sclerosing cholangitis Primary sclerosing cholangitis-autoimmune hepatitis overlap syndrome Secondary sclerosing cholangitis

Primary sclerosing cholangitis Primary sclerosing cholangitis (PSC) is a chronic, progressive disorder of the medium and large ducts in the intrahepatic or extrahepatic biliary tree. Inflammation, fibrosis, and ultimately stricturing of these ducts lead to multiple serious complications including cholecystitis and liver failure. The etiology of PSC is unknown, but PSC is closely associated with ulcerative colitis which leads to speculation of an autoimmune cause. PSC is also more frequently found in men, with a mean age at diagnosis of 40. Patients with PSC may present with fatigue and pruritus, but many are also asymptomatic with incidental abnormal laboratory findings leading to a more thorough workup. Likewise, physical exam may be normal at the time of diagnosis. When symptomatic, likely presentation includes excoriations secondary to pruritus, jaundice, hepatomegaly, or splenomegaly. The most common abnormal laboratory finding is an elevated serum alkaline phosphatase. A subsequent cholangiogram in a patient with PSC should demonstrate characteristic bile duct changes including strictures in multiple locations and dilated segments. The cholangiogram is also important for excluding secondary causes of sclerosing cholangitis, including cholangiocarcinoma, choledocholithiasis, and diffuse intrahepatic metastases. Diagnosis of PSC is made by this combination of demonstrated characteristic bile duct changes and exclusion of secondary causes of sclerosing cholangitis. Additional causes of cholangitis that are important to exclude include immunoglobulin G4-associated cholangitis and PSC-associated autoimmune hepatitis overlap syndrome as their diagnosis changes management. Treatment of PSC focuses on managing this progressive disease and multiple associated complications and slowing or reversing the disease process. Medical management including immunosuppressive and anti-inflammatory agents, as well as endoscopic and surgical approaches, are currently used to achieve these goals, but there are no proven treatments to alter the natural course of PSC. Liver transplantation may ultimately be indicated for advanced disease and the outcomes for patients with transplantation are promising.

A 35-year-old man presents to his primary care provider with complaints of rectal pain, bleeding, urgency, and rectal discharge for the past three days. The patient states that he has recently had unprotected, receptive anal intercourse with a new partner. On exam, the external rectal area is without masses and normal aside from a purulent rectal discharge. Digital rectal exam is significant for diffuse tenderness. Which of the following is the most likely diagnosis? Anal fissure Internal hemorrhoids Proctitis Proctocolitis

Proctitis Proctitis, the inflammation of the rectum, is an acute condition that can be caused by inflammatory bowel disease or infectious etiologies. Receptive anal intercourse is a risk factor for infectious conditions, such as proctitis, anal warts, or perianal abscess/anal fistula. While pain and bleeding are common in many conditions, purulent discharge is suggestive of proctitis. Proctitis presents as a normal visual exam with possible discharge and diffuse pain, and bleeding on digital exam. Infectious causes include chlamydia, gonorrhea, herpes simplex virus, and syphilis. Gonorrhea will likely have a purulent discharge while chlamydia may not. All patients who participate in receptive anal intercourse should be tested for sexually transmitted infections (STIs). STI tests should include anal swabs for gonorrhea, chlamydia, HSV, and peripheral blood for syphilis serology. Anoscopy is recommended to determine if any lesions are present. Treatment should be determined based on the etiology of the condition. Empiric antibiotic therapy for chlamydia and gonorrhea should be administered while waiting for laboratory results. This should include ceftriaxone and doxycycline. Patients who present with perianal or mucosal ulcers, should be treated for HSV infection with valacyclovir, famciclovir, or acyclovir. Patients should return in three months for retesting if chlamydia or gonorrhea are the causative agents. Partners should also be tested and treated for the infection.

The secretion of which of the following physiologically active compounds during menses is thought to be a causative factor in the development of primary dysmenorrhea? Estrogen Progesterone Prostaglandins Serotonin

Prostaglandins Both estrogen (A) and progesterone (B) are produced and secreted from the ovaries and play important roles in the menstrual cycle but are not implicated in the pathophysiology of primary dysmenorrhea. Progesterone is secreted from the corpus luteum in the ovary following ovulation and is important for maintaining the early stages of pregnancy. Serotonin (D) does not have a clear pathophysiologic role in dysmenorrhea but has been implicated in premenstrual syndromes. Selective-serotonin reuptake inhibitors (SSRIs) are considered first-line treatment for premenstrual dysphoric disorder (PMDD) and premenstrual syndrome (PMS) but have no clear role in the management of dysmenorrhea.

A 25-year-old woman presents to the clinic with recurrent breast pain. She states the pain is worse just prior to the onset of menses each month. She denies breast masses or overlying skin changes. Which of the following is the best treatment for the most likely diagnosis? Needle aspiration Oral antibiotics Oral antibiotics and incision and drainage Reassurance and simple analgesics

Reassurance and simple analgesics

What is the most common form of esophagitis in adults? Eosinophilic esophagitis Infectious esophagitis Pill esophagitis Reflux esophagitis

Reflux esophagitis Eosinophilic esophagitis (A) is an immunologic disorder caused by antigen sensitization in susceptible individuals, causing eosinophil-predominant inflammation in the squamous epithelial lining of the esophagus. Characteristic findings on endoscopy include edema, esophageal rings, longitudinal furrows, and punctate exudate. Treatment can include a trial of proton pump inhibitors, elimination diets, or topical glucocorticoids. Infectious esophagitis (B) is most commonly seen in immunosuppressed individuals and can be caused by Candida species, herpesvirus, and cytomegalovirus. Although rare, it occasionally occurs in nonimmunosuppressed patients. Odynophagia (an uncommon complaint in reflux esophagitis patients), chest pain, and hemorrhage can accompany this disease. Pill esophagitis (C) occurs when an ingested pill becomes lodged and does not pass completely through the esophagus. This is most likely due to poor pill-taking habits such as taking with too little water or lying down soon after ingestion. Symptoms typically begin within hours of ingestion and consist of sudden-onset chest pain and odynophagia. Common medications implicated in this disorder are nonsteroidal anti-inflammatory drugs (NSAIDs), doxycycline, tetracycline, ferrous sulfate, and bisphosphonates. Endoscopic findings include localized edema and ulceration.

A 29-year-old woman was diagnosed with cyclic breast pain and initially treated by physical support and ibuprofen. She returns to the clinic today stating that her pain has not improved, and she wants to try other pharmacologic treatment. She is not interested in having children now or in the future. Which of the following is the next best step in the management of this patient? Bromocriptine Fluoxetine Oral contraceptive pills Tamoxifen

Tamoxifen Cyclic breast pain (previously fibrocystic disease or fibrocystic breast changes) is the most common type of breast pain. Cyclic breast pain is caused by hormonal fluctuations during the menstrual cycle and typically causes breast pain during the latter part of the luteal phase. The pain occurs during this part of the cycle because hormones that are released during ovulation stimulate the proliferation of normal glandular breast tissue. Women who present with breast pain shoulder undergo a thorough history and physical examination. Cyclic breast pain is characteristically bilateral and diffuse. It recurs during the same part of each menstrual cycle and is not associated with other symptoms or signs, such as breast mass, overlying skin changes, and nipple discharge. The diagnosis can be made clinically in classic cases. Imaging of the breasts is indicated if the patient is not up-to-date on breast cancer screening or has atypical symptoms, including breast mass and erythema or edema of the overlying skin. The preferred breast imaging (when indicated) for breast pain varies by age. Patients who are under 40 years of age should have ultrasound imaging due to the breast tissue being radiodense in young women. Patients who are over 40 years of age should have ultrasound and mammography. In specific cysts, fine needle aspiration may be both diagnostic and therapeutic. The cysts usually contain straw-colored fluid. There are nonpharmacologic and pharmacologictreatment options for cyclic breast pain. Reassurance that cyclic breast pain is not associated with an increased risk of breast cancer may be the only treatment necessary. The first-line treatment to alleviate cyclic breast pain is physical support (supportive brassieres) and simple analgesics (acetaminophen or nonsteroidal anti-inflammatory drugs [NSAIDs]). Second-line treatment options include tamoxifen and danazol. Although these second-line pharmacologic medications improve cyclic breast pain, they are associated with adverse effects. Tamoxifen causes menopausal symptoms, including hot flashes, vaginal dryness, and arthralgia. Danazolcauses androgenic symptoms, including weight gain, menstrual irregularities, and deepening of the voice. Tamoxifen and danazol use for cyclic breast pain is limited to one to three months of treatment duration due to the adverse effects. Caffeine abstinence and evening primrose oil are treatment options that have not been proven to be effective in placebo-controlled trials. However, they are sometimes recommended because of the low risk of adverse effects and because some women may benefit.

Which of the following most accurately describes choledocholithiasis? The presence of a gallstone obstructing the cystic duct The presence of gallstones within the common bile duct The presence of gallstones within the gallbladder The presence of stones within the genitourinary tract

The presence of gallstones within the common bile duct Choledocholithiasis refers to the presence of gallstones within the common bile duct (CBD). Most cases are secondary to the passage of gallstones from the gallbladder into the common bile duct. However, primary choledocholithiasis, where the stones form within the common bile duct, occurs in conditions such as cystic fibrosis. Patients with choledocholithiasis typically present with a steady and severe right upper quadrant pain, nausea, vomiting, and laboratory testing that reveals a cholestatic pattern of liver test abnormalities. A cholestatic pattern is when alkaline phosphatase (ALP) and bilirubin are elevated relatively more than the liver transaminases aspartate aminotransferase and alanine aminotransferase (AST and ALT). If total bilirubin levels are high enough, then the patient may appear jaundiced. Patients with uncomplicated choledocholithiasis are typically afebrile and have normal complete blood count and pancreatic enzyme (lipase) levels. Complications of choledocholithiasis include acute pancreatitis and acute cholangitis. Acute pancreatitis causes elevated serum lipase and acute cholangitis causes fever and leukocytosis. The first imaging used to confirm choledocholithiasis is an abdominal ultrasound. Abdominal ultrasound is the initial imaging in almost all hepatobiliary conditions. The results of laboratory testing and ultrasound are then used to stratify a patient as high risk, intermediate risk, or low risk for having choledocholithiasis. High risk patients should proceed to endoscopic retrograde cholangiopancreatography (ERCP). ERCP is the gold standard and both diagnostic and therapeutic. In addition, the gallbladder should be removed (elective cholecystectomy) to prevent the recurrence of stones in the common bile duct. Intermediate risk patients may proceed with magnetic resonance cholangiopancreatography (MRCP) to look for stones. If positive, patients should undergo ERCP, followed by elective cholecystectomy. Low risk patients can proceed directly to cholecystectomy without additional testing. The symptoms of choledocholithiasis resolve when the stone passes spontaneously or is removed by ERCP.

Which of the following is the treatment of choice for diarrhea caused by a giardiasis infection? Amoxicillin and clavulanic acid Ciprofloxacin Doxycycline Tinidazole

Tinidazole

A 32-year-old woman presents to the office with a two-day history of bloody diarrhea and painful defecation. She has had about two to three bowel movements per day. She has experienced these symptoms in the past, and her old doctor prescribed suppositories. She does not remember the name of the medication. Upon physical exam, her abdomen is soft, and bowel sounds are normal. She has some mild tenderness to palpation of the lower left abdominal quadrant. Which of the following is the most likely diagnosis? Crohn disease Ischemic colitis Shigellosis Ulcerative colitis

Ulcerative colitis Ulcerative colitis is one of two types of inflammatory bowel disease, the other type being Crohn disease. Ulcerative colitis involves the large bowel only, whereas Crohn disease can affect any part of the gastrointestinal tract. Ulcerative colitis is associated with other extracolonic manifestations such as uveitis, pyoderma gangrenosum, pleuritis, erythema nodosum, ankylosing spondylitis, and various spondyloarthropathies. Other conditions that are associated with this disease are toxic megacolon, colonic carcinoma, primary sclerosing cholangitis and multiple sclerosis. Etiology is unknown, but genetic factors, environmental factors, use of nonsteroidal anti-inflammatory drugs, psychological stress, and dairy consumption may be potential contributors. Smoking may decrease the risk of developing the disease. Patients will present with bloody diarrhea (hallmark symptom), frequent stools, mucus in the stool, and occasionally tenesmus. In severe cases, patients can also develop severe diarrhea, cramping, leukocytosis, and abdominal distension. It is characterized by periods of symptomatic flare-ups and remissions. The disease is categorized as mild, moderate, or severe, depending on the number of bowel movements the patient has per day. Patients with severe disease have more than six bowel movements per day, which can lead to severe anemia, hypovolemia, hypoalbuminemia, and malnutrition. In patients with mild disease, physical exam will be normal or may reveal mild tenderness to the lower left abdominal quadrant. Patients with moderate to severe disease may exhibit abdominal tenderness and cramping, fever, and weight loss. Laboratory findings such as hematocrit, sedimentation rate, and serum albumin are helpful in determining the severity of disease. Stool studies such as fecal blood and leukocytes, ova and parasite studies, viral studies, bacterial cultures, and Clostridium difficile titer should be ordered to rule out infectious etiologies. Diagnosis is made by sigmoidoscopy or colonoscopy. Findings on colonoscopy will reveal marked edema, friability, erosion, and the presence of mucus and pus that is limited to the mucosa and submucosa only. Unlike Crohn's disease which involves the entire gastrointestinal tract, ulcerative colitis only involves the colon, with continuous inflammation extending proximally from the rectum. Colonoscopy should not be done in patients with fulminant disease due to risk of perforation but can be used after the patient has improved to determine the extent of the disease. Treatment is focused on alleviating the acute symptoms and preventing future flare-ups. Initial medical management is with corticosteroids and anti-inflammatory agents (e.g., 5-aminosalicylates such as oral sulfasalazine or mesalazine) in addition to symptomatic treatment with rehydration and antidiarrheal agents. Whereas corticosteroids are important in treating acute disease, maintenance therapy with oral aminosalicylates is the mainstay of treatment to prevent flare-ups. Mesalamine is commonly prescribed depending on severity of the disease and can be prescribed orally or topically (in the form of creams, suppositories, and enemas), with a combination of both routes working better than either route alone. Surgery is considered when medical treatment fails or when there is a surgical emergency (eg., colonic perforation, severe hemorrhage). Total proctocolectomy with ileostomy placement is the only complete cure for the disease.

After a successful vaginal delivery of a full-term infant, a 37-year-old woman has persistent vaginal bleeding leading to postpartum hemorrhage. Which of the following is the most likely cause? Coagulopathy Retained placental tissue Trauma Uterine atony

Uterine atony Coagulopathy (A) can either be a primary (inherited) or secondary cause of postpartum hemorrhage. It can result from a depletion of clotting factors and platelets in a patient with prolonged bleeding. It is not the most common cause of postpartum hemorrhage. Retained placental tissue (B) may contribute to uterine atony and is treated with suction and curettage of the uterus. Traumatic (C) hemorrhage due to lacerations or surgical incisions requires surgical intervention as well as supportive measures, such as the administration of fluids and blood products or tranexamic acid.


Related study sets

Ch. 04 Upper Limb (Hand, wrist, Forearm)

View Set

Insurance License Exam- Ch. 12 Common to Life and Accident and Health Insurance Regulation (STATE PORTION)

View Set

Computer Science Vocabulary Quiz One

View Set

chapter nine: cell communication

View Set

Carrie's Chapter 15: Cardiorespiratory Training Concepts

View Set